Open navigation menu
Close suggestions
Search
Search
en
Change Language
Upload
Sign in
Sign in
Download free for days
100%
(1)
100% found this document useful (1 vote)
2K views
225 pages
AMTI
AMTI GEMS
Uploaded by
Ramkumar Sundaram
Copyright
© © All Rights Reserved
We take content rights seriously. If you suspect this is your content,
claim it here
.
Available Formats
Download as PDF or read online on Scribd
Download
Save
Save AMTI For Later
Share
100%
100% found this document useful, undefined
0%
, undefined
Print
Embed
Report
100%
(1)
100% found this document useful (1 vote)
2K views
225 pages
AMTI
AMTI GEMS
Uploaded by
Ramkumar Sundaram
Copyright
© © All Rights Reserved
We take content rights seriously. If you suspect this is your content,
claim it here
.
Available Formats
Download as PDF or read online on Scribd
Carousel Previous
Carousel Next
Download
Save
Save AMTI For Later
Share
100%
100% found this document useful, undefined
0%
, undefined
Print
Embed
Report
Download
Save AMTI For Later
You are on page 1
/ 225
Search
Fullscreen
from THE MATHEMATICS TEACHER JUNIOR =I THE ASSOCIATION OF MATHEMATICS TEACHERS OF INDIA Son Aan ‘GEMS From THE MATHEMTICS TEACHER JUNIOR -II Edited by Dr. M. Palanivasan THE ASSOCATION OF MATHEMATICS TEACHERS OF INDIA B-19, Vijay Avenue, 85/37, Venkatarangam Street, Triplicane, Chennai — 600 005. Tel: (044)-28441523 E-mail :
[email protected]
Web site: amtionline.comROO OK BETWEEN US Dear Reader, The demand for our books based on NMTC questions had been increasing steadily. Hence it is decided to bring all the problems and solutions of NMTC every five or ten years without disturbing the earlier editions. To satisfy the needs of various age groups they are being brought out in five different books ~ Primary, Sub-Junior, Junior, Inter ‘and RMO and INMO. The Present period covered by these books are from 2004 to 2012. In the case of RMO and INMO it is 2005 to 2013. We hope and trust the beneficiaries of our service will continue to encourage us to help aspirants of excellence in Mathematics Education. We are grateful 10 Smt. K. — Maheswari, Sri G. Gnanasundaram, Dr. M. Palanivasan and Sri V. Seshan respectively for having gone through the pages and edited these books in record time. With best wishes. Yours sincerely, At (M. MAHADEVAN) General Secretary, AMTICONTENTS (iivear2004 — Screening Test Final Test (2stxear/ 20057 Screening Test Final Test 2001 Screening Test Final Test 00 AB mS Screening Test Final Test Screening Test Final Test (2009 Screening Test Final Test Screening Test Final Test Screening Test Final Test qomivear'2012_ Screening Test Final Test 21 61 70 86 95 115 126 141 148 165 174 191 198BHASKARA CONTEST SCREENING TEST - 2004 1. Given the sequence a, ab, aab,cab),aaabb, aaabbb,-»- upto 2004 terms, the total number of times a’s and b's are used from 1 to 2004 terms are (A) 2004 a’s and 2003 b’s ~=—(B) 4008 a’s and t's (C) 1002x 1003 a’s and (1002)?b’s (D) 1003? a’s and 1002x 1003 b's Solution: Looking at the terms of the sequence given, it can be noted that all even members have the same number of a’s and b's while, all odd members have one ‘a’ more than the b's, Further in the n** member, if n is even the number of a’s = the number of b's= 3 and if n is odd, the number of a’s= 2}, while the number of b's= 21. As 2004** member of the sequence corresponds to an even number 2004, the number of a's and the number of #’s in this member will both be equal to 1002: As 2003 is odd, the number of a’s in the 2003"¢ member is 2003+1 or 1002 while the number of 6's will be 23-1 — 1001. -, Number of a’s in the first 2004 members = 1414+2424+34+3+4---+1002+1002 = 2(1-42-4 4 1002) = 2% 2008 * 1098 — 1099 x 1003, Also number of 6’s in the first 2004 numbers OF1F142424343+4+---+ 1001 +1002 Old + 24 +---+ 1000] + 1002 = 2% 200% 100? 4 re99 1002 x (1001 + 1) = 10027. Answer: (C) a6 Problems and Solutions 2., The number of two digit numbers divisible by the product of the digits is (A) 5 (B)8 (0) 14 (D) 33 Solution: Let the number be denoted by ab so that it is equal to 10a+b. As ab the product of tho digite a and b, divides ! (10a + 6); we must have ‘a’ as a divisor of bie. a
0,y > OQ and z > y. The factorsProblems and Solutions 7 of x4 — y* are (c—y), (x+y) and (x? +y"). But these must be 1,3,5 in some order. Clearly c-y
5. fC = 7, then B = 6,D ='8 or Be 8,C =6. If B= 8, then D+ 8. Similarly if B = 6 then 2. But then with a carry, from ten’s place I = 7, a digit ten ‘by C. This is not possible. Hence C % 7. Thus when = 5,C can only be equal to 2, D has three: possible values eh corresponding to which B has the-values 7 or 8, 6 or 8, 6 or 7 respectively. A solution exists only when'D = 7 and B ='8> Problems and Solutions viz., 9827 + 827 = 10654, The digit 3 alone is missing in this equation. Note: We can have a few more solutions such as 9782 + 782 = 10564, 9728 + 728 = 10456 etc. But in all thes cases also the digit 3 alone will be found missing. Answer: (B) Five children each owned a different number of rupees. The ratio of any one’s fortune to the fortune of every child poorer than himself was an integer. The combined fortune of the children was 847 rupees. The least number of rupees that a child had was (A) 12 Rs. (B) 10 Rs. (©) 7 Rs. (D) 5 Rs. Solution: Let the children have a,b,¢,d and e rupees such that they are integers anda >b>c>d>e. It is given that a+btctd+e = 847 and b,¢,d,e are divisors of a; c,d,e are divisors of b; d,e are divisors c; and ¢ is a divisor of d. As e divides exactly a, b,c, d and e, it is a factor of at+b+e+ d+e=847=7x 11x11. In other words e is equal to 7 or 11 or 7x 11 or 11 x 11, But the least value obviously is 7. Hence any child must have had at least Rs.7. Note: A possible solution for a,b, ¢,d,e willbea=b=c=d= 210 and e=7. Answer: (C) . A number with 8 digits is a multiple of 73 and also a multiple: of 137. The second digit from the left equals 7. Then the 6° digit from the left equals (A) 1 (B)7 (0) 9 (D) can be any digit Solution: The number is a multiple of 137 and 73. As 137 and 73 are primes, the number is also a multiple of 187%73 = 10001. The eight digit number which is a multiple of 10001 can be gotProblems and Solutions 9 by multiplying a four digit number abcd by 10001. The product is obviously abcd abcd. The second and the sixth digits from the left are both equal to b. Henée if the second digit is 7 then the sixth digit is also 7. Answer: (B) 8. Let n be the least positive integer such that 1260n is the cube of a natural number. Then n satisfies: (A)l
0 or (n+ 2)? <9. Hence n+.2 = —3,~-2,-1,0,1,2,3. As nis an integer, n can be —5,—4,—3,—2,—1,0 or 1. Hence there are seven possible values for n. Answer: (C) 12. During holidays, five people A, B,C, D and E went swimming regularly. Each time they went, exactly one of them was missing. A went the least number of’times (5 times):and ZProblems and Solutions 11 most often (8 times). What can we say about the number of times B,C and D went? (A) each went six times (B) each went seven times (C) 2 went 6 times and one went 7 times (D) 2 went 7 times and 1 went 6 times Solution: From the given information we observe that (i) F was present on all occasions, (ii) A missed out three occasions, (iii) B,C, .D together missed 5 occasions and (iv) no one except A missed 3 or more occasions. We, therefore, infer two of B,C, D missed 2 occasions while the third one.occasion. In other words among B,C,D two went swimming on 6 occasions and the other went on 7 occasions. Answer: (C) 13. Let A = {a,b,c} and B = {a,b,d,e,f}. How many sets C consisting of characters from the English alphabet can be constructed so that CC B and such that ANC has one element and CZ A. (A) 16 (B) 14 (C)8 (D) 6 Solution: From the given data we infer that C is part of B but not of A and C and A have just one element in common. As ANB = {b} and C is a part of B, ANC also has the element b. Hence C is the set containing b and elements of any subset of {a,c,d,e}. As {a,c,d,e} has 24 = 16 subsets, we can form C in 16 different-ways. Answer: (A) 14, The sum of all angles except one of a convex polygon is 2190°. (where the angles are less than 180°). Then the possible number of sides of the polygon is (A) 13 (B) 15 (C) 17 (D) 1912 Problems and Solutions Solution: If the convex polygon has n sides, we are given: that the sum of the angles of the polygon except one = 2190° = 12 x 180° + 30°. As the sum of all the n interior angles is (2n—4) right angles or. (2n— 4) x § = (n—2) x 180°, the angle left out must be 150°, the supplement of 30°. Then sum of the interior angles = 13 x 180°. a=2=Worn=15. Hence the given: polygon has 15 sides. Answer: (B) 15. In aright angled triangle with legs 4 and 8 , the area of the largest square that can be inscribed in the triangle is (A) § (8) 3 (©) # (2) P A Solution: Let the square inseribed be one with side a. t ‘Then NA area AABC'= area. AAFE + { e t area AEDC ei + area of square BDEF ie, bx 4x8 = ME) 4 MED 40? ie, 32=Waora=}. Hence the side of the inscribed square Answer: (A) 16. Two circles with centers A and B and radius 2 touch each other externally at C. A third circle with center C and radius 2 meets the other two at D, E (see the figure).. Then area ABDE ig (A) 3v% (B) 6V3 (C) 3V5 (D) 6v3Problems and Solutions 13 Solution: -Clearly AAEC' and ACDB are both equilateral and congruent with sides equal to 2 units. ADCE is also an equilateral triahgle with each side equal to 2 units. So, ED = 2 unite. But AB = AC + CB = 4 units. ‘Area of thé quadrilateral ABDE (in fact,; this-is.a trapezium) ia’ equal to sum: of the areas of the identical equilateral triangles AGE, ECD and CBD of sides equal to 2 units. Hence, the required area = 3 x 4 x (48 x 2) x 2 = 3V3 square units. : : " Answer: (C) 17. In AABC, ZA =.90° and J is the incentre:‘The perpendicular distance of I from BC is ./8.’Then AI is equal to (A) v8 (B)3 (©). viz (D) 4 Solution: Let the incirdle touch the sides: BC, CA, AB in P,Q,R respectively. “As mZBAC = 90°, AQIR is a square of side = JR = in radius =r, say. As IP =r'= V8, (given), AI the diagonal of the ‘square AQIR is IRx V2 = V8v2 = 4 units. Answer: (D) 18. ‘In an isosceles triangle, ‘the centriod, the orthocentre, the incentre and the circumcentre are (A) conincident (B) collinear (C) in the interior of the circumcircle (D) it the interior of the incircle Solution; AABC is iyosveles with LB = ZC. Let AD be the altitude from A. The orthocentre H lies on AD,.Also AD14 Problems and Solutions bisects the vertical angle BAC and so it contains the incentre I, Further AD bisects perpendicularly BC. Hence it contains the circumcentre S and being a median it also contains thé centroid G. Hence H,I,S and G are collinear, Answer: (B) Note: These four points will coincide when’ AABC is equilateral. When A is obtuse H lies outside the circumcircle as well ss the incircle. Thus (B) is the only correct answer. 19. If a,b are positive real numbers and \/a% = a.,/§ where af is a mixed fraction, which of the following is true? (A) b=o? +1 (B)a=t-1 (C)ash4+1 (D)b=a?-1 Solution: The value of the mixed fraction af is a + $. ‘Therefore the given condition implies. ab+1)_ a? = a+ S =a" x Fie. . b b+1 =a? (Cancelling § on both sides, a > 0,6 > 0). Answer: (D) 20.2 +$+E =1and $+2+£=O then the valueof 2+ +5 is (A) 0 (B) -11 (C9 (p)1 Solution: Writing s = 2,y = } and z = £, we are given that atytz=1and 244 41=0. The second equation implies yz + 22+ cy = 0, Now, apy te? = (ety +z)? ~%Ays+ 20 t zy) P-2x051 ; iP ogot Bm : ie. ate ae 1 Answer:(D)Problems and Solutions 15 21, Let [x] denote the greatest integer integer less than or equal to z, what is the value of [VT] + [V2] + [v3] +++» + [V2004]? (A) 58850 (B) 59730 (C) 59950 (D) 56718 Solution: We have (m+ 1)? = m? + 2m +1 60 that the integer parts of Vmi?, /m? + 1, Vm? +4, -++ Wm? + 2m will all be m so that [Vm] + (Vm? +1] + ++. + [Vm™ + 3m = (2m + 1).m = 2m? +m, Also 2004 = 1936 + 68 = 44? + 68. (V48] + (V4d2 41] + +: + [VE + 6B] = 69 x 44 Hence the value of the required sum = SS Va + (VmPa A +--+ Va + 60 44 mat = Com? +m) +69 x44 met = 2has(as + 1)(2 x43-41) 4 Be = 43x 44x 29 +43 x 224+ 69 x 44 = 22x [2404 + 43 +198] = 22 x 2675 = 58, 850. Answer: (A) ox 44 22. If the roots of the equation 2” —2axr-+a?+a—3 =0 are real and less than 3 then (A)a<2 (B)2
4 Solution: From the given equation, we get (x ~ a)? = 3—a or z =a+ /3—a, As the roots are real 3—a > Oora <3. As the roots are also less than 3, we have a + /3—a <3 and a- 3-4 < 3.From a+ V3—a < 3, we get V8-a < 3-0 This implies a # 3, Hence a < 3. As 3—a can be written as V3aV/3—a and V3—a > 0, we get 1 < VS—a. Hence a is not greater than or equal to 2. In other words a < 2. When @<2,a~ /3—a <3 is certainly true. Answer: (A)16 Problems and Solutions °93. If a function f(z) is defined such that 10/©).= 4:2 where x <1, then f(a) + f(6) is equal to (A) £88) 8) £(#88): (> £(#5%) (D) none of these Solution: f(a) and f(b) are defined only if a <1,b< 1. ya) ine o) Lob 1G) = TE and 1) = Hence rof@+s70 _ (=a) (1~d) i * (+a) (1+) (1+ ab)—(o+8)_ 1-(285) _1-s . OF aby + (a+d) ~~ 1+(@8) Ive atb = 19/7 =4 = 10/@) where z Tyre provided x ‘<1. Now, = th
at+b- ie. if (1 —a)(1 ~ 6) >.0, which is true ds a <1, <1. F(a) + £() = F(z) = FCPS). ‘Answer: (A) 24, How many solutions are there for (a,b) if 7a673 is a five digit number divisible by 99? ; ‘ (3. Ba (yo ()1 Solution: We can write 7ab73 as Zab hundreds:+73. But this is same as 7ab times 99 plus 7ab+ 73. For 99 to be a divisor of 7ab73, we therefore have 7ab+ 7S a& a multiple of 99.” Giving various digital values to a,b forin 0,1,2,+++,9, we find that 7ab+ 73 can be a number from 700+73 to 799+73 i.e.,Problems and Solutions 17 number from 773 to 872. In this range only 792 is divisible by 99. So, 7ab coincides with 792 — 73 i.e., with 719. Therefore, a = 1,6 = 9. Thus there is only one pair of values (1,9) for (a,b). Answer: (D) 25. The number 107°° — 76™ is divisible by (A) 61 (B) 62 (C) 64 (D) none of these Solution: If-a,b (a > 6) are real numbers a” —)” is divisible by a—b for integral n > 1. Now 107% — 76% = (1072)** — (76?)® is divisible, by 107? — 76? i.e., by (107 + 76)(107 — 76) i.e., by 183 and 31. ie., by 3,31 and 61. The option (A) contains 61. Answer: (A) 26. A sequence a9,a1,02,03,--- ,@n--+ is defined such that a = a, = 1 and any: = (an-1.0a) +1 for n > 1. Which of the following is true? (A) 442006 (B) 3.22009 (C) 5lez00 = (D) 2 f 2008 Solution: We have anj1 = On@n-1 +1, a9 = a = 1. 02 =2, ag =3, a4=7, 5 = 22, ag = 155, a7 = 3411,--- Expressed congruent modulo 4 the sequence is 1,1,2,3, 3, 2,3,3,--- This is a cyclic sequence from a2 onwards, the numbers 2,3,3 cyclically repeating. Therefore ag9qq = 3(mod 4). So, 4 does not: divide agoo4. (If it is a divisor, ag994 must be congruent zero modulo 4). Similarly expressing the sequence congruent modulo 2, modulo 3 and modulo 5, we get respectively (i) 1,1,0,1,1,0,1,1,--» congruent. modulo 2 (i) 1, 1,2,0,1,1,2,0,-++ congruent modulo 3 (iii) 1, 1,2, 3, 2, 2, 0, 1,1, 2,3, 2,2,0,-++ congruent modulo 5. From (i) azoq3 = 0(mod 2)... 2 divides agoo3- From (ii) a2g03 = 0(mod 3). .*,3 divides a2003-18 Problems and Solutions From (iii) a2004 = 2(mod 5). .*. 5 does not divide azo04 exactly. Thus (A) alone is true. “Answer: (A) 27. A solid cuboid has edges of length a,b,c. What is the surface area? (A) (a+b-+ 0)? — (a2 +0? +02) (D) ab+ bc+ ca Solution: Surface area of the cuboid (from the figure) = 2(ab+ be+ca) = (a+b+c)?— (a? +6? +7). Answer: (A) (B) abe (C) 2(a? +B? +c?) a 28. A circle and a parabola are drawn on a piece of paper. The number of regions they divide the paper into is at most (A) 3 (B) 4 Solution: The parabola can cut the circle in a maximum of four points. When the parabola does not touch the boundary of the paper as in the adjoining figure, we get the number of regions as 6. However when the boundary of the paper touches the parabola region II of the figure is further divided into 2 regions - I(a) and II(b). ° ‘Thus the maximum number of regions got is 7. (O)5 LY 2A\ js @) 6 x7 Note: The options do riot contain the number 7, but have onlyProblems and Solutions 19 the number 6. Assuming that the paper is sufficiently large that the boundary is not a tangent. to the parabola, option (D) is correct. 29, A-cubic polynomial P is such that P(1) = 1,P(2) = 2, P(3) =3 and P(4) = 5, Then P(6) is (A) 7 (B) 10 (©) 13 (D) 16 Solution: Let p(z)=a+bx+er?+dz* Then a+2b+4e+8d (ii) +3b+9¢-+27d- (iii) a+4b+16c+ 64d (iv) (ii)—(i) gives 1=b+3c+7d (v) (ii)-() gives 1=b+5e+19¢ (vi) (iv)—(iii) gives 2= b+ 7c+37d (vii) (vi)-(v ) gives “0=2c+12d (viii) (vii )-(vi) gives 1=2c+18d (ix) (ix)—(vill) gives. 1 = 6d or d= $. «'. (ix) gives. 1=2e+3 orc=-l. w. (¥) gives —8+horb=¥%. v.() gives L=at+ 2-14 $ora=-1. Hence p(x) = —1+ 42-27 + 325. <. p(6) = —1+ 2 x 6 — 36 + § x 216 =-1417-36+36 = 16. Answer: (D) 30. Which of the following is the best approximation to (23 = 1)(3? - 1)(4* - 1) (+188 +1) +1) (A) § ® % (C) 8% (©) S0020 Problems and Solutions Solution: Note that m—1 = (m-1)(m?+m+)) m +41 = (m+1)(m?-m+l) mtm+1 = (m+1P?-(mt)t1. . Given expression : ' (1890-1) (1000?-+-1000-+1) $$1)(10007—100041 As 274+24+1=37-341,37+34+1=4—4+ 1 etc, after cancellations, the given expression = }-3-2-+- @-40ogyssope __ 2 (1000000 + 1000+ 1) 1000 x 1001 x 3 = 0.666001332 ‘The given options are equivalent to 0.6, 0.66. 0.666 and 0.6666. Hence the expression is nearest to 0.666. ‘Answer: (C) uate a) Amicable Numbers ‘Two natural numbers in and n are called, Amicable (or friendly) if m is equal to o(n) and n is equal to o(m) where o(k) denotes the sum of all the proper divisors of k. For example, 220 and 284 are amicable numbers. If two numbers dre Harshad numbers and are also amicable, they are called Amicable Hershad numbers. (2620, 2924) is one such pair. The Wonder World of Kaprekar Numbers ~ Page No.68-69.BHASKARA CONTEST FINAL TEST - 2004 4. Show that there are no integers a, b,¢ for which a? +H -8c=6 Solution: For any integer n we always have w= 014 (mod 8). So the’ different possibilities for a? + 6? modulo 8 are 0+0=0, OF1=1, 141=2, 14+4=5, 044=4, 444=0 But a? +8? — 8c = 6 implies that a? + 6? = 6(mod 8) which is impossible as per the above observation. Hence there are no integers a, b,c for which a? + ? — 8c = 6. 2. Given that N = 2"(2"+1 — 1) and 2°+? — is a prime number, show that a) Sum of the divisors of N is 2N b) Sum of the reciprocals of the divisors of. N is 2. Solution: Let 2"+? —1= P. Then we are given that P is a prime number. We have 2"? = P+1 and 2" = 2. This makes N = 2" x P. We find that the divisors of 2" x P are 1,2,22,23,..+ ,2",.P, 2P,2°P,29P, ++ ,.2"P. So the sum of the divisors of N is piven by on = 1t2e Pepe” +P(L $2427 +23 4006 +-9) (P+1) Eps *) (P+1)(" -1) = artgntt _ 1) = 2N This proves (2). il i 2122 Problems and Solutions Remark: One can also use the formula for the sum of the divisors on of a positive integer n given by ith pet oan | +L oy pitt ~1 on = (Pat) (eet os (Et pa-l -1- - aode Pr where n = phipl? ... pit ...pk is the prime factorization of n. ue ni P?- *) (fi 2-1 )\ Pri tt —1(P +1) 2-an(ant} — 1). 2N. on uo We Maen alee seo a= once : 1 Py = lf rata tele +5 +R ate ‘t+ 5l = i baled “+3 Ay+ i 1-Gyr) | (a 1-$% - (art —1) 7 gat Ra—1) * GTI) = 2 This proves part (b). 8. Given three non-collinear points A, B,C construct a circle with center C such that the tangents from A and B to the circle are parallel. Solution: Construction: Find the mid-point M of AB. Join C to M and draw lines AX and BY parallel to MC. From CProblems and Solutions 23 drop perpendiculars to AX and BY to meet them at P and Q respectively. Draw the circle > with center C’ and radius CP. Lis the required circle, Proof of Construction: Since AM = MB and AX||MC|\BQ we have CP = CQ. Now AX and BY are parallel tangents to the circle). 4, Given a circle with diameter AB and a point X on the circle different from A and B, let ta; t and tz be the tangents to the circle at A,B and X respectively. Let Z be the point where the line AX meets tb and Y be the point where the line BX meets t,. Show that the three lines YZ,t; and AB are either concurrent or parallel. + Solution:24 Problems and Solutions As AB is a diameter, we have ZAXB = 90°. Let ZX AB = 6 and ZXBA = ¢. We note from the “angle in the alternate segment” theorem that LXAP = $, LAXP = 6 = LZXQ, 2BXQ=0=ZYXP ZAYB = 0, ZAZB = $,2XBQ = 6 In the right angled triangle AXY, we have PA = PX = PY. Therefore, P is the mid-point of AY. Similarly from ABXZ we get QX = QB = QZ, Therefore Q is the midpoint of BZ. Also AY is parallel to BZ. Hence YZ,PQ, AB concur at a point. If X is the mid-point of the arc AB, we see that the lines YZ, PQ, AB are parallel. 5. The polynomial ax? + bc? + cz +d has integral coefficients a,b,c,d. If ad is odd and bc is even show that at least one roat of the polynomial is irrational. Solution: Suppose the roots 2), %2, 23 of az? + ba? +er+d=0 are all rational numbers. Then the transformation y = az transforms the given cubic into o(2)'+0(2)" +e(%) +d=0. ‘This is equivalent to y+ by? + acy +a7d=0. () ‘The ronts of this equation are Vi = ALi, yo = 29,43 = arg.Problems and Solutions 25 e Tt is clear that the roots y1,y2,y3 are all rational numbers. ‘As the coefficients of equation (1) are integers, we must have ‘yt, yo, y3 a8 integers. Further these integers 1,92, ¥3 satisfy utytys=—b (2) yiye + yays + yay = ac @) nyrys = —a7d. (4) Now, in view of equation (4), yi, ¥2,ys must be divisors of a?d. Since it is given thet ad is an odd integer, 41, 2, ys are all odd integers. From equations (2) and (3), it is clear that b and ac must be odd. This means thet a,b,d are all odd integers. ‘Therefore, equation (3) implies that c is an odd integer. Now, b and ¢ are odd contradicts our hypothesis that bc is even. So the given equation has at least one irrational root. Let f be a function from N to R satisfying (o) #(1) =1 and (b) #(1) + 2f(2) + 8f(8) +++ + F(R) = n(n + F(A). Find (2004). Solution: We have £(1) + 2f(2) + 3f(8) +--+ + OF (a) = n(n + YF) $(1) +2f(2) +3/(8) +--+ +(n-1)F(n— 1) = (n—1nf(n—1) for n > 3,-Then subtracting the second equation from the first, we get n?f(n) = n(n—1)f(n-1) or (n=1)f(n—1) = nf(n), forn >3 Sy 2f(2) = 3f(3) = 4f(4) = ++» = nf(n) for n > 2.26 Problems and Solutions From (b) with n = 2004, we get F(1) + 2F(2) +--+ + 2003. f(2003) = (2004)? F(2004) or 1+ (2002) x (2004) F(2004) = (2004)? f(2004) 1 3004(2004 — 3003) L = 08 «*. (2004) a 7. Consider @ permutation pipspapapsps of the six numbers 1,2,3,4,5,6 which can be transformed to 123456 by transposing two numbers exactly four times. By a transposition we mean an interchange of two places — for example, 1 2 3 4 5 6 to 3 2145 6 (positions 1 and 3 are interchanged). Find the number of such permutations. Solution: We observe that any permutation of {1,2,3,4,5, 6} can be reached in at most 5 transpositions. For any permutation P given by pipapspapsps we count the number of pairs i < j such that p; > pj and call it o(P), Then every transposition applied to P changes the (odd-even) parity of c(P). This means that the set of all permutations can be separated into those reachable by an odd number of transpositions and those reachable by an even number of transpositions. Any permutation reachable in 0 or 2 steps can be also be reached in four by repeating the same transposition twice. So we are just required to count the number of even permutations. Fix a transposition T. Then T defines a correspondence P — TP from the set of even permutations to the aet of odd permutations, which is clearly one-one. This means that the number of even permutations is one half of theProblems and Solutions 27 total number of permutations, which is 6! = 720 in our case. So the answer to our question is 360. 8. Let 01,09,43,'-+ ,@m be a sequerice of real numbers. The sum of k— successive terms is called a k—- sum, for example a; +aj41+0;42-+++++0;44-~1.i8 a k— sum. In a finite sequence of real numbers every 7-sum is negative and every 11-sum is: ypositive. Find the largest number of terms in such a sequence. ‘Solution: We writé the successive 7-sums in separate rows: If we continue writing till the first 11 such 7-sumis, we get 11 rows of 7-sums; which in turn gives 7 columns’of 11-sums. See the adjoining table. aytag+agtess+az < agtagtaat stag < agtagrastesstag < < < <= oon coco ii +412 +043.. +017 < In the above table each row sum is. negative and so the sum of all the terms is negative. Qn the other hand, each column sum is positive and go the sum of all the terms is positive. This leads to a contradiction. Therefore at best we can have 10 such 7-sum rows. This means that. the sequence can have at most 16 terms. By triel and error method we construct the sequence of 16 terms 5,5, —13, 5, 5, 5, —13, 6, 5, —13, 5, 5,5, -13, 5,5 satisfying the given requirements.BHASKARA CONTEST SCREENING TEST - 2005 1. (22, 48), (61, 76), (29, 34) are some pairs of distinct two digit numbers whose product ends with digit 6. How many such pairs are there? (A) 47 (B) 315 (0) bag (D) 405 Solution: All two digit numbers that end with 0 or 5 do not contribute (since a product of these will always end with 0 or 5,not 6). 11 multiplied by 16,26,--» ,9§ account for 9 numbers 12 multiplied by 13,23,---,93 and. also by 18,28,---,98 account for 18 numbers. Similarly 13,14,16,17,18 and 19 these account for 6x9 = 64 numbers. In total where one of the numbers lies between 10 and 20, we could locate a total of 10 x 9 = 90 pairs. This can be repeated for pairs with one of the numbers lying between 20 and 30,30 and 40,:-- and 90 and 100. ‘Thus total number of pairs will be 9x 90 = 810. However, since we have considered all pairs, the number of unordered such pairs will be only half of 810. «. No, of distinct pairs = 99 = 495. Answer: (D) 2, The digits 1,2,3,4 are used to generate 256 differant 4-digit numbers. The sum of the 256 numbers is (Ay 71440 (B) 711040 (C) 704110 =~ (D) 741040 Solution: In the units place each of the digite will occur aS = 64 times 28Problems and Solutions 29 In the ten’s place each of the digits will occur 445 = 64. times In hundred’s place each of the digits will occur 28% = 64 times In thousand’s place each of the digits will occur 2§8 = 64 times ", Total = G64(14+2+8+44)+64(10)(1+24+3-+4)+64(100)(14+24+3-+4) +64(1000)(1+2+3+4). = 640+6400+64000+640000=711040. Answer: (B) o Find the area in square centimeters cf the shaded rectangle if all other shapes 1,2,3 in the large 9em x Sem rectangle are squares. (A) 4 (B) 3 (C)2 (D)15 Solution: Area 2 is @ square of side 5cm and area 25cm?. Similarly area 3 is a square of side 4cm and aréa 16cm?. Area 1 is a unit cm square with area 1em?. Hence the area of the shaded rectangle is 3em?. (Note that 9x 5--4x4—5x5-1x1=45- 42 =3). ‘Answer: (B) 4, In the diagram PS = PQ and QS = QR. If ZSPQ = 80° then . ZQRS equals (A) 15° (B) 20° (C) 25° (D) 30° * ' Solution: Since PS = PQ,
equating the integral and fractional parts on both sides, we get w= and oy = fy ast = & implies SH = Boor c+ i=o+}. Hence’ r=9 and }=} or z=9,y=2. Thus w=5,c=9,y=2 so that wto+y=5+94+2=16, Answer: (A)Problems and Solutions 37 20. PQRS is a common diameter of the three circles. The area of the middle circle is the average of the arcas of the other two. If PQ =2 RS =1 then the length of QR is is (A) 146 (B) V6~1 (C)4 (D)3 Solution: Let diameters PQ,PR and PS be D;,D2,D3 respectively. Then "74 = 5 [F (D} + D3) > 2D} = D3 + Dj. Given D, = PQ =1;D3 ~ Da = RS = 0.5 nD} a 2(D3 — 0.5)? =1+D3 or D?-2D;~0.5=0. oD; = BF 12 8. Length QR = Dz—Dy = (D3—0.5)-1= Dy-15 =-34.%8. As QR is not negative, QR = =fv8, Note: None of the given options gives the answer Answer: (=14¥8) , 21. The positive numbers x and y satisfy ay = 1. The minimum value of 2y + a is (A) $ (B) § (o)1 (D) no minimum Solution: The problem is to minimize f = Jy + zy subject to 2y=1 or to minimize f= a+ + %. For minimum jf, £ =0 = —de-$ +29 = 28 =4 or at =2. Also £4 = 202-6 +32? > 0 for z>0. <. f hes a minimum at the positive 4th roots of 2. .. Minimum value = $+4%2=1. Answer: (C)38 Problems and Solutions 22. Of the points (0,0), (2,0), (31), (1,2), (3,3), (4,3) and (2,4) at most how many can be on a circle? (A) 5 (B) 3 (4 (D) 6 Solution: Denoting the seven given points by O, B,C, D, EB, F and G, from the rough sketch we see that all the points except F are at the same distance V5 units from D. (Note that if the figure represents a chess board “a horse” placed at D can ina single move go to O,B,C,E or @ but not to F). Thus the five points O,B,C,E,G are concyclic. The equation to this circle is (x — 1)? + (y-2)7=5. Answer: (A) 23. How many of these expressions 2° + y',a‘ + y3,23+y? and z*—y‘ are positive for all possible numbers = and y for which z>y?. (A) i (B) 0 (C)2 @)3 Solution: Construct the following table: (= > y).. zly[@+v [ety [ery late of-] + - - - eet + _ a +{-] + + + + +{4] 4+ + + + +40 ia + + + From this table one can see that for all values of z,y with z>y, only z°+y¢ is positive. Answer; (A) 24. ‘The number of ordered pair of digits (A,B) such that A3640548981270644 B is divisible by 99 is (A)3 (B)2 (1 (D) zeroProblems and Solutions 39 Solution: As the number is to be divisible by 9 and 11,. (i) sum of the digits is divisible by 9, and (ii) difference between the sum of the odd digits and the sum of the even digits is-a multiple of 11. : ‘Thus we have 71+ A+B is a multiple of 9.or A+ B-1-is a multiple of 9 and (37 + A) ~ (84+ 8) is divisible by 11. This A+B = 1,10 or 19 and A—~B = ~3. Obviously A+B #19 as A and B are single digits. If A or B is zero then the other, can be one but the difference will not be 3. The remaining possibility is that A+ B= 10 and A-B=~3. ‘This is also not possible since if A+B is even, then A—B must also be even. No values of A and # exist. Answer: (D) 25. The roots of the equation 2°—402!+P23+Qz2?+4Rr+S=0 are in geometric progression. The sum of their reciprocals is 10. Then [S| is equal to (A) 16 (B) 32 (Cc) 4 (D)1 Solution: Let the roots be ar*,ar,a,%, 4. ‘Then sum of the roots = a(r?+r+1+24 4) =40. The sum of their reciprocals = 4(3 +1 4+1+r+r?) = 10 dividing one by the other a? = 4 or a= 42. Product of the roots =.a° = (+2)5 = +32 =-S. |s| =32. + Answer: (B)BHASKARA CONTEST FINAL TEST - 2005 1. p(z) is a polynomial of degree n and p(k))= > for #=0,1,2,-+,n. Determine p(n +1). Solution: We observe that (k + 1)p(k) — k = 0 for all & in {0,1,2,---n}. So we consider the polynomial g(z) = (z+ 1)p(x) — « and we see that the polynomial q(x) has the following n+1 numbers 0,1,2,--- ,n as roots. Therefore 9(z) = Kz(x—1)(x—2)(2-3)---(e~n) for some-constant K. ‘We evaluate (x) at —1 to get a-1) = -(-1) a = K(~1)(-2)(-3)--- (-n-1) = K(-1)"**(ng1)! Thus K = coma As p(2) = set we get pny = ant ietos) nt t+ Ye (nt in\(n—1yin— n+2 — mti+(-yn ii n+2 Therefore, 1 ifn is odd Pint 1) = aa ifn is even 40Problems and Solutions 41 2. Two sides of a triangle are V3cms and V2cms. The medians to these sides are perpendicular to each other. Find the third side. Solution: Let AD,BE and CF be the medians through the vertices A,B,C - of F AABC. Let BE and CF E be the given perpendicular medians. Using cosine formula B c AB? = AD? + BD? ~2A4B.BDcos ADB. and AC? = AD? + DC? — 2AD.DC cos ADC. But cos ADC =~cos ADB and BD = DC = 480. Hence adding, we get AB? +40? = 24D? +.2(2 |AD? = 2AB? + 2AC? — BC?. Similarly 4BE? = 2BC? +2AB? — AC? and 4CF? = 2AC? + 2BC? — AB?. As G divides each median in the ratio 2:1 we get BO)?. BG? = tpEt = yeas? ‘+280? - CA), oot = foe = j@AC* +2047 45%), Since BEC = 90°, BG? + 0G? = BO’. at jean? +280? — CA?) + gee? +2CA? — AB?) = BC? or 2AB? + 2B0? ~ CA? +. 250? + 26.4? — AB? = 9BC?42 Problems and Solutions or BC? = AB? +CA?. But AB = V3 cm and CA= v2 cm. SBC? =34+2=5 cm? or BO =1om. Hence the third side BC is of length lcm. O js the circumcentre of triangle ABC ond K is the circumcentre of triangle AOC. The lines AB, BC’ meet the circle AOC again at M and N respectively. L is the reflection of K in the line MN. Find the angle between BL and AC. ° 7 A L 3 B Solution: B (Case 1 Case 2 Case 3 These are the three possible configurations. Case (i) 4MCB = £MCO+Z0CB = £MA0+ZOBC <4MBO + ZOBC 4MBC wl Case (ii) ZMCB = Z4OCB-ZocB 4OBC —- ZOAM ZOBC~ZOBA <£MBC "Problems and Solutions 43 Case (iil) 2MCB = ZMCO~ZOCB (180° ~ ZOAM) — ZOBO 180° - ZOBA ~ ZOBC Z4MBC ‘Thus in all the three cases ZMBC = ZMCB. We now, show that L is the circumcentre of AMNB. Case (i) ZMLN = 4MKN = 2Z£MCN = 2ZMBN Also LM = LN. Therefore L is the circumcentreof AMNB. Cese (i) ZMLN = 4MKN 2£MAN 22MCB 2£MBN w As LM = LN, L is the circumcentre of AMNB. Case (iii) 4MLN = £MKN = 24MCN 2Z4MCB 2ZMBC = 360°- 2MBN tt Again using the fact that LM = LN we sce tliat as in the other two cases here also L is the circumeentre of AMNB. So, finally 4MLB = ZMNB = 2ZBAC This gives ZMBL = 90° — ZBAC. This means that BL and AC are perpendicular.44 Problems and Solutions 4, Show that the equation z?-+y?-+ = Seys hay infinitely many solutions in positive integers. Solution: We may write the given equation as y+o2—s(Byz-2) = 0 ory? +274 (8yz—2)? — Syz(3yz—z)=0 or y? +2? + (3yz—2)? = 3y2(3yz—=2) ‘This shows that if 1 <<
Byz—2(0222) = 3e-2(3 < 3y) = Ww>z So the new solution y,z,3yz— satisfies y << 3yx—a and the new solution set has a bigger biggest element. Starting with the solution set 1,1,1 by repeated application of the above process, we get (1, 1,2), (1,2,5), (2,5, 29), (5, 29,433), -+ as some other integral solutions. Thus the given equation has infinitely many solutions in positive integers. 5. Find all pritae numbers p and g such that p(p+1)+9(¢+1) = n(n+1) for some positive integer n. Solution: p(p +1) +4(¢t+I) = n(n +1) > p(pt1) = (n—g)(n+q41)- ‘This means that p divides n~g or n+q+1. If p divides n—q,then n—q>p. Therefore, nt+-qg+l=n—q+2g+1> n-qtl>ptl. But this will contradict p(p+1)+9(q+1) = (n—g)(n+q+1). So p has to divide n+9+1. This means that n+q+1=kp for some positive integer k and p+1 = k(n—g). We note that (p+a)(e+a+1) = r(p+ 1) +9(9 +1) + 2pg>n(n+1) > p+q>n. Without loss of generality we may assume that p >. As n(n +1) > p(p+1) we get n> p. Now, n+qg+l=kp>n>Problems and Solutions 45 p3k>1. Also, kp=nt+q+1<(ptg)+g+1<3p+1< 4p k <4. So k has to be 2 or 3. Suppose k= 2. Then n= 2p—q—1 and p+1=2n—2g. This implies that 2n = 4p — 29-2 = p+1+2q and therefore 38p—3 = 4q. Hence 3 divides the prime number g which implies that q = 3. This gives p = 5 and n = 6. When k = 3, we get n = 3p—q—1 and p+1 = 3n—3g. This gives 3n = 9p —3qg—-3 = p+1+3¢ and therefore 8p—4 = 6g. This gives 3n = 9p—3q—-3 = p+1+3q and therefore 8-4 = 64. ‘This implies that ¢ = 2,p=2 and n=3. Thus, the solutions are given by (p,4,2) = (8,5.6) or (5,3,6) or (2,2, 3). 6. Show that there is no integer valued function f on the set of integers such that f(m+ f(n)) = f(m)—n for all integers m™m, Tr. Solution: From the given functional equation we get f(m + 2f(n)) = F(m + f(n)) = frm) —n where m, = m+ f(n). So, we get H(m+2f(n)) = f(m)-n= oe - = f(m)—n—n= f(m)- ‘We prove by induction that f(m+kf(n)) = f(m) — kn for all positive integers k. We assume the result for k—1 and observe that f(m+ kf(n)) S(m + (k—1f(n) + f(r) fm+ (k~ 1)f(n)) —2 f(m) —(k-3)n—n = f(m)— kn Q) ‘The above result is vacuously true for k=.0. Again, f(m) = f(mn - f(n) + f(n)) = fim — f(n))~n tw w uProblems and Solutions f(m— f(n)) = f(m) +n = f(m)-(-1)n This means that the result (1) is true for k = —1. We now prove by induction that the result (1) is true for all negative integers. We assume the result for —(k — 1),k > 2. f(m ~ kf (n)) 1 f(m— (kU f(n) - f(r) f(m=(k-1)f(n)) +n f(m) -(-(k-1)n+n f(m) + kn. il il a Hence for all integers k we have f(m + kf(n)) = f(r) — kn. As (1) is am integer, (1 +£(1)/(1)) = f(1)- f) =O. Let K =1+f(1)f(1). Then K > 0 and f(K) = 0. For any integer n we have f(n) = f(n+f(K)) = f(n)=K < f(n) which is impossible. Hence there is no such function f. Proye that it is possible to partition the set of positive integers into 100 non-empty subsets Aj, Ag,-++ , Aigo so that if a +2005 b=c for integers a,b,c then a,b,¢ are not all in different Aj. Solution; Define f :N 4% by f(n) = largest k such that 2 divides n. Define Aj = {n|f(n) =i mod 100} for i = 0,1,2,3,-+-99. Let a,3,¢ be integers such that a + 2005b = c. Case 1: Suppose f(a) < f(6). In this case 2005 6 is divisible by @ higher power of 2 than a since 2005 is odd. So the highest power of 2 that divides ¢ must be the same as the highest power of 2 that divides a. In other words f(a) = f(c).Problems and Solutions 47 Case 2: Let f(a) > f(b). In this cose it is clear that 7(b) = f(c). So, in either case, two of a,b,c have the same f -value; which means that two of a,b,c always lie in the same Aj. 8. The set A consists of 100 positive integers less than 200. Prove that there exists a non-empty set 9 such that the product of all its elements is a perfect square. Solution: For any subset C of A, we can.write the product of all its elements as (C) = c?Po where c? is the largest square that divides %(C) and Po is-elther 1 or product of distinct primes each less than 200. Since the number of non- empty subsets of A is 2° —1 which is definitely greater than 2* where & is the number of distinct primes less than 200. Now, ‘for any subset C of A, Po is a product of at most k distinct primes and so the number of possible values of Po is at most 2*. As we have more number of non-empty subsets of A than the number of possible values of Pc there must be two non-empty subsets X,¥ of A such that Py = Py. If B is the set (X —Y)U(¥ —X) then since X'4 Y we see that B is non empty. Further each number in XMY appears twice in the product P(X).B(¥). Hence PP yPPy = 2y?(Px)? BXYP(Y) B(B)B(X NY)? The above equation implies that PB (B) is a perfect square. uBHASKARA CONTEST SCREENING TEST - 2006 1. Tn an n sided regular polygon the radius of the circum-circle is equal in length to the shortest diagonal. Then n is (A)6 (B)8 (C)12 (D) Such a polygon does not exist. Solution: Let A,B and C be the three consecutive o vertices of the n-sided regular polygon and O be the centre of its circum circle. It is given that the shortest diagonal is equal to the radius of the circum circle. In the figure, AC’ is the shortest diagonal AC = OA= OC = ZAOC = 60° = ZAOB = 30°. So the number of sides of the regular polygon = 98 = 12. The answer is C. pa . All the six faces of a cube are extended in all directions. The number of regions into which the whole space is divided by these 6 planes is (A) 9 (B) 16 (0) 24 (D) 27 Solution: The cube has 3 pairs of opposite faces. Correspondingly we get 3 pairs of parallel planes. The first pair of parallel planes divide the space into 3 regions, ‘The second pair of planes divide each of these 3 regions into 3. We get 9 regions each of which is further divided into by the third pair, We get 27 regions, The answer is D. 48 yProblems and Solutions 49 3. Let A=ata?+a2+--- isa (A) positive integer (B) negative integer (C) negative fraction (D) positive fraction Solution: A=-141-141---141=0 AAP =0 a negative integer. The answer is B. 4, How many possible values can one get by multiplying 2 different numbers from the set {4, 8, 9, 16, 27, 32, 64, 81, 243}? (A) 12 (B) 24 (c) 36 (D) 48 Solution: >i[ei[r [sls] # PB? | PP | ZS" | 73* [ aes? | 2a | Da! | Daa? 2x3? | hed? | 2x3" | 243° 2x3? [2 [2h 2x3* 283? | 263? | 2x3" | 238 zy 2| 2 ye] 2 | 2 | 20 3? | 3%x2? | 3x2? | 3%x2" B | 392 | 3x2? | 3x2" | 372? | x2" | 3* 3H | tz? | 3x2 | 342" | 342° | 340%] 3° BP | 3x2? | 3%x2? | 3%x2t | 3*x2° | 3%x2* | 3” | 3 —t 2, 2", 3! and 3” are not products of different numbers. Table50 Problems and Solutions ‘The numbers in the set are 27, 2°, 24, 25, 28, 37, 39, 34,35. There are 3 types of products: (i) 2.25 (ii) 3°.3°, (ip 293° . In (i) 243 < a+b < 5 +6; the products are 25, 28,27, 28, 29, 210, and 211. We have 7 products. In (ii), 243 Sa+b< 445; the products ars 3°,3°, 37,38 and 3°. We have 5 products. In (ili), 25 a <$6,2<5b<5. We have 5x 4 = 20 products. The total number of products is 7-+5 +20 = 32. The problem is to be corrected with one choice as 32. 5. Define a*b = 2a+2b- ab. For example, 4*3 = 2x4+2x3—-4x3=2. If 3e2=2ec, then z is (2 (B)1_— (C)O.— (D) No such z exists Solution: 3xr=6+22—-32=6-2 2ee=4+22—-Ie=4. Since 3x2 = 22, we get 6—c=4->2=2. The answer is A. 2 . If ABCD = (ABC) x (BD), where A,B,C and D are digits not necessarily distinct, then the value of C is (A)O (B)1 ~~ (C) any digit (D) any nonzero digit Solution: If B= 1 and D =0, then the product (ABG) x (BD) = (ABC) x 10 = ABCD. So C can have any value. It is enough if we choose B,D properly. The answer is C. 7. Mf axb=a+b+] and if ax5=b*4=11, then asd is (A) 10 (B) 11 (C) 12 (D) 13Problems and Solutions 51 Solution: a+5=a+6,b%4=b+5 ‘> axdb=5%6 = 12. “The answer is C. 8. a,b and care three natural numbers. Exactly two of them are odd."‘Then (A) a+b+c+ab+te+ca is odd (B) ab+be+ca is even (C) abe is odd (D) a? + be is even Solution: a+b+c is even. If a and 6 are odd, then ab is odd and bc + ca is even, therefore ab + be + ca is odd. Thus a+b+c+ab+be+ ca is odd. Clearly B is false; so is C. a? + bc is odd as one of a? or be is odd. Thus the only true statement is A. 9. Given z+3y = 100 where x and y are positive integers. The number of pairs satisfying the above equation is (A) 100 (B) 97 (C) 34 (D) 33 Solution: 2+3y= 100 > y= 334432. 2 =1,4,7,---97. ‘There are 33 values of zhence there are 33 pairs of solutions (x,y) for 2+ 3y = 100. Thus the answer is D. 10. A three digit number with digits A,B,C in that order is divisible by 9. A is an odd digit and C is an even digit. B and C are non zero, The number of such three digit numbers is (A) 20 (B) 16 (C)8 (D4 Solution: See (page 14) Subjunior Qa No 6 of Kaprekar Contest. The answer is A. 11. The number of prime numbers lesa than 1 million whose digital sum is 2 ia, (A) 5 (B) 4 (c)3 (D) none of these52 Problems and Solutions Solution: Similar to Qn No 7 of Kaprekar Contest. The answer is C, 12, The value of "on simplification is a @! @& © ©) (a)” Solution: (36)% (22 x 92) ae SKS _ mex git 3 12 = Fionscge = 3a" - = (3) = (5) and the answer is D. 13. In Eldorado, the baso in the number system is b, unlike our decimal system where 235 means 2:10? +3-10+5. The place values in Eldorado are powers of } and the currency is in dinars. Ram gave a 1000 dinar bill for a bicycle costing 440 dinars and receives 340 dinars as change. The base } is (A) 12 (B) 8 (7 (D) none of these Solution: 1000 =1x 08 +0-6?+0-b+0 and 440 = 4 x U3 + db + 0,340 = 30? + 4b-+0 40? + 4b-+0+30? + 4b +0 = 05+ 087+0-b+0 => 7b? + 8b = BS => 7-7-8 =0 as 6 #0 => (b- 8)(b-+1) =0,b= 8 or b= —1, But b=-—1 is not possible . b= 8. The answer is B.Problems and Splutions 53 14. A certain number leaves a remainder 4 when divided by 6. ‘The remainder when the number is divided by 9 is (A)lor4 (B)40r7-(C)lor7 = (D) 1, 4or7 Solution: Let the number be n so that n= 6k +4 for some (positive) integer k or n = 2(3k +2). Now, k= 1 n= 10> n= 1(mod 9), k=2=n=16>n=71(mod9), K=33n=W>ns 4(mod 9), k= 4 n = (mod 9), & = 5 > n = 7(mod 9) and k= 6 => n= 4(mod 9). Thus the possible remainders when n is divided by 9 are 1,4 zs and 7. The answer is D. Note: 6k +4 for any & € N cannot give remainders other than 1,4 or 7, 15. Given the alphametic where each ABCD letter represents a different digit, -BCDA the number of distinct solutions is ivi (A) 0 (B)1 (C) 6 (D)7 Solution: See (page 12): Sub Junior Qn No 1. We find that A=1 and A= 2 are not possible. If A = 3 we can have D=5,C =6 and B=1. If A=4 then D=6,C =7 and euae Similarly, A = 5,6,7,8,9 ate possible, Thus we have 7 solutions, namely, ABCD = 3165,4276,5387, 6498, 7609, 8720, 9831. The answer is D. 16. Ina MABC, ZA = 30°, AC = 10 units and BC = 4 units. Which of the following statements is correct? (A) ZB is acute’ (B) 2B is obtuse (C) 4B is go° (D) such a triangle does not existSolution: In any triangle 324 = a2p- Here a= 4,b= 10. =>sinB= » > 1, not possible. So, such aA does not exist. The answer is D. 17. The number of solutions (x,y) where z and y are integers, satisfying 2x? + 3y? + 2x + 3y = 10 is (A) 0 (B) 2 (C4 (D) none of these Solution: The given equation can be written as Az? +24+4) +3? +y+4) = 104343. ie., 2(22 +1)? + 3(2y +1)? = 45. This first of all shows that (2x + 1)? is divisible by 3. Since (22 + 1)? is the square of an odd integer and is divisible by 3 and 2(2x +1)? < 45, we see that (2c +1)? = 9. Hence (2y +1)? =9. Thus, z= 1 or —2,y=1 or —2. We get 4 solutions (1,1),(1,—-2),(—2,1) and (-2,-2). The answer is C. 18. BC is the diameter of a semicircle, . ‘The sides AB and AC of a triangle ABC meet the semicircle in P and Q respectively. PQ subtends 140° at the centre of the semi- circle. Then ZA is oe (a) 108 () 20" (C) 30° (D) 40° 6 ¢Problems and Solutions Solution: ZPOB + LPOQ + ZQOC = (180 — 2y) + 140 + (180 - 2z) => 2x + 2y = 320 r+y= 160. In QABC.Z4B +20 = t+y = 160°. ~. ZA = 180 — 160° = 20°. The answer is B. 19. In rectangle ABCD, ZC is trisected by CE and CF where E lies on AB and F on AD. If BE = 6cm and AF = 2cm, which of the following integers is nearest to the area of the rectangle F ABCD in sq.cm.? (A). 130 (B) 150 A (C) 170 (D) 190 Solution: In ACEB, 4ECB = 30°. “CB = V3 x 6om. “FD = 6V3 — 2cm. 5556 Problems and Solutions oi c d > x6 4 LY E B Area of rectangle ABCD =CDx BO = V3(6V3-2) x V3x6 = 36(3V3 - 1) = 36x (5.196 - 1) = 36x 4.196 = 151.056. So the answer is B. 20. The first two terms of a sequence are 0 and 1. The nth term Tp = 2Tm-1 — Tn-2,0 2 3. For example the third term Ty; = 21, —-T, = 2-0=2. The sum of the first 2006 terms of this sequence is (A) PocoxzooT —(B) 2005x2006 (C) 2006 (D) 2005 Solution: The first 3 terms are 0 and 1 and 2. The next few terms 2x 2~1=3, 2x3-3=4,2x4-3=5. Ih general if any two consecutive terms are a and a+1, then the succeeding term is 20+2-a=a+2. Thus we get the sequence of whole numbers 0,1,2,3,-+ 2005, 2005 being 2006th term. Sum of these 2006 terms is 205X206 and the answer is B. Note: The given relation can be written as Ty, — T-1 = Tn-1 ~ Tn-a- Thus, T,72,73,--- is an A.P. The first two terms of which are 0,1. So T, =n—-1.Problems and Solutions 57 21. Eighteen balls are placed in 4 boxes so that no box is empty. Then (A) At least one box has at least § balls (B) At least one box has exactly 5 balls (C) Exactly one box has at least 5 balls (D) Exactly one box has exactly 5 balls Solution: There are 18 balls. If we place 4 balls in each of the boxes first, there are 2 balls left out. Both these balls can be put into the same box. So that one box has 6 balls. This distribution rules out the B, and D. If the remaining two balls are put in two boxes, then two of the boxes has 5 balls each. This choice rnles out C’ and D. Therefore the answer is A. 22. The least number of numbers to be deleted from the set {1,2,3,+++ 18,14, 15} so that the product of the remaining numbers is a perfect square is (A) 4 - (B) 3 (c)2 (@)i Solution: See (page 15) Qn No 10 of Kaprekar Contest. The answer is B. 23. If (43), in base 2 number system is equal to (34), in base-y number system then the values for z and y ate respectively (A34 (B) 4,3 (0) 7,9 = (D) 9,7 Solution: (43)e = (34)y > Ao + 3 = By +4. = 40 —3y=1 ‘The pairs in A,B and D do not satisfy this while the pair (x,y) in C satisfies, So the answer is C. 24. Consider the following sequence: a1 = minimum {aj-1.a;.2} for i> 2, Then a2o0s (A) 1003 (B) 1002» (CG) 1001 (D) none of these. 2 qs lt58 Problems and Solutions Solution: It is easy to find that the sequence is 1,1,2, 2, 3,3, 4,4,--- 1003, 1003. Thus the 2006th term is 1003, The answer is A. > 25. In the adjecent figure BA A and BC are produced to meet CD and AD produced * y e in E and F. Then ZAED+
Area of APS = 28q unit by (1) ® r ie Area of AAPQ = 3q unit. (2)60 Problems and Solutions Area of AARC = 4 times area of AASQ =4 square’ units [Sq || RC and SQ = }RC}. (3) Area of AABC = 4. Area of AAPQ = 12 square units. (4) Area of trapezium PBQCG = Area of AABC — Area AAPQ = 12~3=9 square units. (5): Area of that SQRC = Area of AARC— Area of AASQ = 4-1=3. (6) , Area of trapezium PSRB = 9-3 =6 square units. The answer is A. 30. A solid cube is chopped off at each of its 8 corners to create an equilateral triangle with 3 new corners. The 24 corners are now Joined to each other by diagonals. How many of these diagonals completely lie in the interior of the cube? (A) 36 (B) 96 (G) 108 (D) 120 Solution: The 24 corners seen be joined to form ™4%28 — 276 line segments. Some of them are edges, some are face diagonals and the remaining are interior diagonals of the solid. The total number of edges is 12+8 x 3 = 36. The top bottom and the four lateral faces are octagons. An n sided polygon has (2) - diagonals. So each of 6 faces contains (8) —8 = 20 diagonals. The triangular faces have tio diagonals. So the total number of face diagonals is 6 x 20 ="120. The remaining 276 — 36 — 120 = 120 segments are interior diagonals. The answer is D, Note: See (page 46) Qn No 26 of Ramanujan Contest for an alternate method of solution.BHASKARA CONTEST FINAL TEST - 2006 tl. In AABC, ZA = 75°, 2B = 60° and ZC = 45°. Also CF and AD are the altitudes from C and A respectively. ie His the orthocentre and O is the circumcentre. Prove that O is the incentre of ACHD. Solution: Given ZA = 75°, ZB = 60°, ZC = 45° in AABC. CF and AD are altitudes from ©,A respectively. H is the orthgcentre and O is the circumcentre of AABC’. ; Required to prove: O is incentre of ACHD. 6162 Problems and Solutions 4FHD = 180° — ZB = 120°, 08 B,F,H,D ate concyclic. «. CAHC = 120° .. 2DHC = 60°; ZFHA = 60°. (Linear pairs of CFHD.) 1 4PAH = ZHGD = 90° (Sum of angles of A is 180°). & e . CHAC = 75° — 30° = 45° and ZHCA = 45° — 30° = 15°. AADC is isosceles with AD = CD ZA = 2C = 45° in AADC. ~. Perpendicular bisector of AC passes through circumcentre O. But is also angular bisector of D. «DO bisects ZADC_or ZHDC. QQ) ‘at+y= 60° ytz=48? >) r= 45%, y= 15°, 2= 30°. zt+a= 75° .. [CO_bisects ZHCD |, (2) Hence O is incentre of AHCD, (from (1) and (2)).Problems and Solutions 6S —1— show that atb+c 3 i 1 1 Bets eters 1 1 Solution: Given t4iet c atbt+e™ => (a+0)[P+ca+d)+ab] =0 (a+) -(c+a)-(c'+6)= =a Jor => [e=-bor [e Consider a= —b (or) 1 1 1 gts aa Symmetrically for c= —a (or) b=—c, we get A+a ta u . @° BS S+R+S Thus it is proved. 3, Let 1
[S=as1 =a] (Due to telescopic cancellation of terms @2,a3,-"* 50) - S is maximum when 51 is maximum end a; is minimum. ~'. Smax = max(asi) — min(a1) . But as; < 142 and is natural = max(as1) = 141 and a; >1 and is natural + min(a;) = 2. Snax = 141 — 2 = 139. Let us assume that among the 50 consecutive differences a; —ai-1 for i {2,3,4,--» ,51}, no value occurs 12 or more times. Then S$ is minimum when a difference of 1 repeats 11 times exactly, a difference of 2 repeats 11 times exactly, a difference .of 3 repeats. 11 times exactly, a difference of 4 repeats 11 times exactly and a difference of 5 repeats 6 times exactly. _ (+ltit---+1) (24+ 242+---+2) “8 bead 11 times 11. times (84+34+3+---+3) + (4+44+44---44) 11 times 11 times (5+54+5+++-+8) 6 times Smin = 11 x (1+2+3 +4) +5 x6 = 140. Sin > Smax contradiction. Hence our assumption is wrong. There is some value of difference that occurs at least 12 times. Hence proved.Problems and Solutions 65 4, Prove that in eny perfect square the three digits immediately to the left of the unit digit cannot be 101. (For example: --: 1012 can not be a perfect square) Solution: ‘Any perfect square ending with digit 6 only will have an odd digit in the tens place. Proof: Let any whole number be represented by 10z+y where y is the last single digit and 2 can have any number of digits. (10x + y)? = 1002? + 202y + y?. (102 + y)? = 2? hundreds + 2zy. tens +y. Examining all the possible values of 17? we see that the digit in the tens place of this is odd only when digit in the units place is 6. <. Ia perfect iquare ends with 101z, then x must be 6 only. oN = +--1016 is 8 perfect square. But last 3 digits suggest N is divisible by 8, since 016 is divisible by 8. And last 4 digits suggest that N is not divisible by 16, since 1016 is not divisible by 16. But a perfect square if divisible by 8 = 2 x 2 x-2 should be divisible by 16 = 2x 2x 2x2. As N does not satisfy this condition, N cannot be a perfect square. 5. S= [r+ By) + [r+ ie) + [r+ fol ts + [r+ Hl = 1606. where'r is a real number and [z] denotes the greatest integer not exceeding . Find [100r]- Solution: If z
+ | i 20 and {fr +2 mie 21, sort Biota < 2100 < 100r + 94, 2006 < 100r < 2007. So [100r] = 2006. , Given that c,b,¢ are positive real numbers such that a? +b? + = 3abe. Prove that pfx + abs + aie 2 abr: Solution: Given a,b,c,€ Rt and a?+¥+c? = 3abe Required to prove a b c 9 sst+as 2>-— wat aa? aR oybte @ ++ 9 ae — atbt+e > (atbte)(a +8 +03) > ae? = (Babe)? e ee (atbteoj(ad +h tc) > (2? +R +02)? (2) It jg enough that we prove (2) to prove (1). By Cauchy- Schwartz inequality, we get [vay + (75) + (Va?) . [Cova)? + v8? + (v8 > (Vala.va) + Vb(bv8)) + Veleve)?. or (a+b+c).(a5 +8 +c3) > (a? +H? + 2)? Thus (1) is proved.Problems and Solutions 67 7. Find all integer solutions (a,c) of a4 + 6a? +1la?+6a+1= g(a? ~ 1)(? = 1) a+ negative powers of alternate primes ie. g = 2511. where 6; >0. where q is the product of arbitrary non- Solution: If a = 0(mod 3), then a’ + 6a? + 11a? + 6a +1 = 1{ mod 3). If a = 1(mod 3), then a4 + 6a? + 11a” + 64+ 1(mod 3) . Mh a* +1la? + 1(mod 3) 1411 x 1+41(mod 3) 1( mod 3) til ut If a = 2fmod 3), then af + 6a? + 11a” + 6a + 1fmod 3) ul a‘ + 11a? + 1(mod 3) 1411 x 1+ 1(mod 3) 1{mod 3) at WW .. Invariably, L.H.S of the equation ot 4 bet + thet 46041 = = ME 1) a+ is always 1(mod 3), where a,c are integers and g is product of arbitrary non-negative powers of alternate primes. ie. g= 251109... where b; >0. For the equation to be true, R-H.S of the equation should be Lmod 3). Reframe the equation as (a? + ?).(a* + 6a? +110? + 6a + 1) = g.(a? - 1). — 1) (1)Problems and Solutions When both a, are Ofmod 3), then LHS of (1)= Ofmod 3) where as RHS of (1)= 1 or 2Gnod 3). When a,c are not simultaneously Ofmod 3), then. LHS of (1) = 1 or 2(mod3) whereas R:HS of (1) = Ofmod 3). In both cases, the equation (1) is invalid. °, There are no integral solutions in (a,c) for the given equation. Find the maximum positive real number k such that +h < 2 ‘or all positive real numbers 2! VG tVIGe Ty) ~ & and y. Solution: Given ——— VG + Bz? + y?) where z,y € Rt for some ke Rt. < ie => (8x7 + y?)(a? + y?) > Ke?y? =O G+ BP (or) 2 2 3414+ 3544 28. Foe eee 4 ew a +(vaz 4) +2V3 > k, ( B= ‘v 2 + (V5 +1)? > angProblems and Solutions 69 But LHS > (v5+3)?. -- (vag—#)° 20. Equality holds when ¥ = 93. As z,ye Rt. Equality holds when LHS = (v3 + 1)? and Bis correspondingly maximum. Rac = VB+1 Alternate solution: Consider 1 1 (92? + 9°) + a) 2 @) Now by Cauchy Schwartz inequality, we have 2 1 1 (ove) + or] [7+ 2] ; ah oe > (vie +y2) =(v8+1)2. [vier + or [Gedy] 2 sen, (302 + 9?) +4) 2 (VE? @) From (1) & (2), we get ‘max = V3+1, since ke Rt.BHASKARA CONTEST SCREENING TEST - 2007 1. Ine polygon there are 6 right angles and the remaining angles are all equal to 200° each. The number of sides of the polygene is (A) 15 (B) 12 (9 (D) 23 Solution: Let the number of sides of the polygon be n. Given 6 x 90° + (n ~ 6) x 200° = (n — 2) x 180° => 3x 180° + (n —6) x 200° = (n — 2) x 180° => (n—6) x 200° = (n— 5) x 180° = 10(n—6) =9(n-5). Solving, we get n= 15. Answer: (A) 2. Two natural numbers differ by 41. The bigger number is greater than 30 times the smaller number plus 10 The smaller number b : (A) 11 (B) 2007 (C) 71 (D)1 Solution: Let the natural numbers be z, (41+ 2). Given 41+2>302+10 > 31> 292. Only z= 1 satisfies 292 < 31. Answer: (D) 3. The value of, (1 ~ gk) (1 ~ gh) (1— fe) ++ (1 gghpn) is (A) ite (B) 380% (C) 3005 @)1 70Problems and Solutions 71 Solution: (-3) 2°2°3°3°4°4°" 2007 _ 1. 2008 _ 1004 * 2" 2007 ~ 207° Answer: (B) P Q 4. PQRS is a rectangle of m B, area 2000cm?. Two lines L parallel to the sides are D c drawn to cut the rectangle s R into four rectangles of : ae fc Q ares A,B,C,D, Given A = 100m? , B =< A aiee le 5o0em?, C = 300m’, | tt D = 200cm? which of the * 7 —" following is possible? : ; { z (A) Such a division is not possible (B) Exactly two such divisions are possible (C) At least two such divisions are possible (D) In such a division the point L lies on one of the diagonals Solution: Given A = 1000cm? B = 500cm? C = 300cm” D = 200cm?72 Problems and Solutions Denote the parallels drawn to the eldes by XY and MN. Let PX=2, SN=y ond NR=z (00 the figure). Then zy = 1000 (1) az = 500 (2) wy = 200 (3) wz = 300 (4) 5. If (z,y) is a solution set of the system of equations zy = 8 and wyt+yototy = 54, then 2+y?= (A) 62 (B) 46 (C) 20 (D) 100 Solution: Given zy =8; zy(z+y)+2+y = 54 ie. (cy+1)(e+y) =542+y=6 1 oto? = (e+y)? - Qzy = 6? -2x8=20 Answer: (C) 6. The number of natural numbers n for which Matint# ig a naturel number is (A) 8 (B)2 (©)s (D)4 Solution: Let N = Jaliint$ where Nn are natural numbers, ie, N=15n+8+ § isa natural number.Problems and Solutions 73 So, £ is a natural number =n is 8 positive factor of 6. Hence ni can take values of 1,2,3 and 6. Answer: (D) Tf a,b,c are real; a#0,b40,c#0 and atb+c#0 and b+ hth = she then (at b)(b+e)(c+a) = (A) 1 (B) 3abe (C)o (D) abe Solution: Given a,b,c are real; a #0; 640; c#0; at+bt+c¢0; 24+} +4=-5h5 oe 1 1 a*b ~ arbte ae pe c—a-b-c > ath = datbto eee eee ab. e(a+b+c) => (at+d)[e+cla+b)+ab] = 0 => (at b(c+a)(c+b)=0 Answer: (C) 8. In a rectangle the length is x units more than its breadth . If its length is increased by y units and its breadth is decreased by z units, the area is unaltered. The breadth of the rectangle is ws @ 2, © SE ~ (S%) Solution: Given 1 =b+2; (I+ y)(b-—z) = 1b * ub—-2z)-(b+a)250 = bly — 2) ~ ay +2) =0 (or) b= SABE Answer: (A)74 Problems and Solutions L 9. Two circles touch internally at A. AOB is the diameter of the bigger‘, EN circle of centre 0. OPQ isa (KW tangent tothe smaller circle. APL is a straight line, If ZBOL = 42°. then ZLOQ = (A) 84° (B) 104° (C) 90° (D) 17° Solution: Let C be. the centre of the smaller circle. @ 4 Zcap = 3x42 =21 ZPCO = 21° 421° = 42° cP 180° —(90° + 42°) => (QOL) = 90° Answer: (C) 10. The units digit of 32007 x 72008 , 132009 5, (A) 3 (B)1 (9 (D)7 Solution: 377 = (34)501 x 38, (34)01 results in 9 value ending with digit ‘1’. Hence 3°? results in a value ending with digit ‘7’. 77008 — (74)504 74 results in a value ending with digit ‘1’. Hence 77° results in a value ending with digit ‘1’, 137009 — (134)502 x 7131. (134)502 results in a valte ending with digit ‘1’. Hence 1379 ends with digit ‘3'. J. The end digit of 37007 x 72008 x 132009 will be the end digit of (10x +7) - (10y + 1) - (102+ 3). for some natural numbers z,y and z. This number has 1 in the units digit. Answer: (B)Problems and Solutions 75 11. In the adjoining figure, AB = AC. DEF is an equilateral triangle. Then (A) a+b+c= 180° (B) a+b=2 (0) a= He (D) ate=2 Solution: Let ZABC 4ACB =< as AB= AC. By exterior angle property, we get : 60 +a=c+e 60° +b=a+e a- -a => Ja=b+e. Answer: (C) 12. a,6,¢,d are four ‘distinct positive real numbers such that a>b,c
c and d
candd
b, b>c > a>b>c e
a>d>c => a is greater than b,c and d. It is impossible to say whether b> d- (or) 6
3- Veed (1) ctd+a>3-Vede (2) d+a+b>3-Vdab (3) atb+c>3: Vabe (4) From (1), (2) , (3) and (4), we get (b+¢+d)(c+d+a)(d+a+b)(a+b+c) >81-abed SA ei g@acaior limi son Thus the minimum value required is 81. Answer: (C)Problems and Solutions 77 15. In the adjoining figure the diameter of the circle is 2007em. . Then c= T (A) ftom 8) em snl (C) 2007cm ~e A (D) 3 x 2007cm | Solution: After simplifying the figure, we get 3r = 2007 + = where 7 = 202 => ¢ = 2 Answer: (A) 16. The number of real solutions of the equation 1+2+427+23 = ai+a5 is (A) 1 (B)5 (C)4 (D)3 Solution: 1+2+2?+23 =2t +25 (1+2)+ (142%) =2-(1+2) (l+2)- (24-27-11) =0 (1+) - (424 — 427-4) =0 (L+2)-{(207 - 1)? - 5) =0 2+1=0iec=-1 Huu78 or (22? -1P = an? -1 = It 227 Problems and Solutions 5 v5 v5+1 Hei [vo+1 (or) if 22 =-—J5+1, x has no real solution. «. There are 3 real solutions in x. Answer: (D) 17. A two digit number is increased by 20% when its digits are reversed. Then the sum of the digits of the number is (A)2 (B)7 (c)9 (D)8 Solution: Let the tens placed digit and the units placed digit values be x,y respectively, for the 2-digit number. . Given : y+ ie. loy+a 50y + 52 => d4y = > dy = where 120% of (10a +-y) Foe +y) 602 +6y 56a 5a & € {1,2,3,4,5,6, 7,8, 9} ¥ € {0,1,2,3,4,5, 6, 7, 8,9} ‘The only solution is z = 4 and y=5. ‘The sum of the digits is 4+5=9, Answer: (C)Problems and Solutions 72 18. If a,b, c,d are positive integers such that a = bed,b = eda, c= abd and d= abe, the value of (a-+b-+c-+d)*/(ab-+be-+cd+da) is (A) 16 (B) 1 (C) 34 (D) # Solution : For solution,see at the bottom of page No 173 Answer: (A) 19. a,b,c are three numbers satisfying the following conditions. (i) abe #0 (ii) 0+b+c= abe (iii) (0+ 5)(b + c)(c+a) £0 and (iv) #24 + pte + gt2'= kabc, then k= “(Ayo (B)1 (C) -1 (D) 2 Solution: a+b+¢= abe a+b = c(ab~1) + axe =-c Be bte cta Similarly, we get [j= =a and $7" = —» ~ ath bte . eta “Treb tint Fine Hence k=—1. Answer: (C) Note: ab or be or ca cannot take velue 1. If ab = 1, then a+b+e=c=>a+b=0 (or) b=~a forcing a? = —1, which has no real solution. «, (1 — ab), (1 ~ be), (1 — ca) cannot take value zero. 20. The sum of the fourth powers of the roots of the equation a - 2? ~22+2=0 is (A)1 (B)5 (c)9 (D) 13 Solution: 2° - 2? - 22 +2 = 0 > (2? -2)(2~1) = 0 soz, 1 ©. The sum of the fourth powers of the roots of the equation is (/2)* + (—V2)4+ 14 =44+44+1=9, Answer: (C) =—(a+b+c) = —abe,80 Problems and Solutions 21. If tan9 and tang are the roots of the quadratic equation 2? Ar+B = 0 and cot@ and cot¢ are the roots of 2?—-Cr+D=0 then CD= (A) AB ®) as © & Os Solution: Given tan8,tang are roots of the quadratic equation: 2?- Azr+B = 0 = tand+tand = A and tan@- tang = B. Also, Given ‘cot@,cot¢ are roots of the quadratic equation: z?—Cz+D =0 => cot0+cot¢ = C and cot ®- cot =D. Now, C.D = (cot@+cot¢)-cotd-cotd 1 4 1 cou (aap eatg) eaves _ tang +tané A = (and-tangy ~ BF i il Answer: (C) 22. A,B,C can walk at the rates 3, 4, and Skm an hour respectively. ‘They start from a place at 1,2,3 hre. respectively. ‘When B catches A, B sends him back with a message to C’. At what time C gets the message? (A) 5hrs. 1émts. (B) 3hre.30mts. — (C) 6hrs.16mts. (D) Shrs.45mts. Solution: Let B take ‘t’ hours to meet A, 2 sta BQL +t) St=3. .. At 5hrs, B mects A and sends A with a message to pass on to C. By thia time, C would have travelled (5 — 3) x 5km = 10km whereas B or A has travelled 3x 4= 12km, ‘The gap between A and C is 2km and let them meet each other efter ‘k’ hours. => 3k-+5k=2 => k=} hour, . C received the message at 5 bra 15 min. Answer: (A)Problems and Solutions 81 23, The units digit of (1+ 9+ 97+ 99-+ +++ 970) is (A)1 (B) 9 (c)o (D) 8 Solution: For a natural number ‘n’. 9” ends with digit 1) Lif nis even 9 if nis odd <9" +9") ends with digit ‘0’, whether n is odd or even. Now 1+9 ends with digit ‘0’, as1 +9 = 10 97+93 ends with digit ‘0” 9 ‘ 9% ends with digit‘ | @) g2006 + 97007 ends with digit'0’ (L494 97 49% +--+ + 97008 + 92007) ends with digit ‘0’. Answer: (C) 24, The sides of a triangle are integers. The perimeter is 8. The area is (A) V8 (B)8 (C)5 (D) 12 Solution: The only possible triangle will be of side lengths 3cm, 3cm, 2cm. Semi perimeter $ = 4cm. By Heron’s formula, area of triangle VAG 24 - 3)(4 = 2)cm? 2V%em? Answer: (A)62 Problems and Solutions %.IE r+ y+2= 2007, syt+y2ten=4011 c #1; yx], z#1 then the value of sista) +h; is (A)1 (B) 2008 (Cc) (D) 0 Solution: (1-y)Q-z)=1-(yt2)+y2 (1) (l-2)Q-y)=1-(e+y)+2y (2) (l-2)l-2) =1-(¢+2)+22 (3) 2)+ (2) + (8) + (1-9-2) +A 21-2) + (1-2)(1-y) =0 (4) because 3—2(2-+y-+2)-+2y-+yz+ 20 = 3—2x 2007 +4011 = 0 (4) +(1-2)(1—-y)(1-z) assuming that 2 #1; y #1; 2 #17! 1 1 Wace iy 1 +7=B 70 Answer: (D) 26. A boy multiplies 423 by a certain number and obtains 65589 as his answer. If both the fives are wrong but the other digits are correct, the right answer is (A) 63489 (B) 60489 (C) 62689 (1D) 60689 Solution: 60089 = 423 x 142 + 23, Verify by long division. +', 60089 + 400 = 423 x 142 +23 + 400 (or) 60489 = 423 x 143, Answer: (B)Probleme and Solutions 83 27. Let A’ denote the ratio of the volume of a cube to that of the sphere which will fit inside the cube. Let B be the ratio of the volume of a cube inscribed in a sphere to that of the sphere, then (A) 4? = 278? (B) B? = 2742 (C) A? = 9B? (D) B? = 9A? Solution: When cube is inscribed in a sphere, diameter of sphere = diagorial of the cube. When sphere is inscribed in the cube, diameter of sphere = side of the cube. 3 A= BaF 0 oe . 4n(pe)s ® > wl > =3v3 => A? = 27B?, Answer: (A) 28. If the average of n observations (where n is odd) arranged in ascending order is a, the average of the first "1 observation is b and that of the last "f* observations is c, then the "$1 th observation is (A) (nt 1)(b-+e)—na a) Metered) (n+1)(o+¢) - 2na a+b+c oe () =* Solution: Let n = 2m+1 Given sum of all observations = na ie S=(2m+1)a84 Problems and Solutions Sum of the first ("$4) observations = Sum of the first (m+1) observations =(m-+1)-6= 5 Sum of the lest (*$4) observations = Sum of the last (m+1) observations = (m+ 1)-c= S2 ‘The last term of the first ("$1) observations is the same as the first term of the last ($2) observations as ‘n’ is odd. th GE) observation = (m-+1)"* observation = H+H-S = (m+1)(b+e)—-(2m+1)a = 2B o+e)—na (n+1)(6+0) — 2na mena coe Answer: (C) 29. The ratio between a two digit number and the sum of the digits of that number is a:b. If the digit in the units place is n more than the digit in the tens place then the number in the tens place is n(a—b) n(a+) (A) T1b— 20 @) 11b- 2a 2n(a — b) (@-») . ©) Tp ©) aie aj Solution: Let the unit placed digit and the ten’s placed digit of the 2-digit number be z,y respectively.Problems and Solutions i Wy+z 4 Given y=2—n; “HEF = § ldy+ytn a fe > Fyn 7% (8) Wily +n) = al2y +n) = y(11b— 2a) =an— tn (or) _ na ¥= Te Answer: (A) 30.-For the equation @— = —6) _ (2-o)(z—d) z—a-b z—c—d Med) — ella +) the only solution ab(e~d) — cdl0 +8) i. one of the solutions (A)z= (B)z= ed (C)z= SPOT ETS ig the only solution (Dye OAD E FD is 5 solution Solution: Let z—a—b=8; r—c-d=t. The equation reduces to ex Hlere) a (t+ ae +o) and further reduces to ab ed, bc +d) ~ eda +8) g-a-b z-c-d + a-a@ Answer: (A)BHASKARA CONTEST FINAL TEST. - 2007 1. Given AABC where AB = AC and ZA = 80°, a point M is taken inside the triangle such that ZMBC = 30° and ZMCB = 10°. If the angle bisector of ZBAC meets BM extended in N, then (1) prove that ABNC is isosceles and (2) hence or otherwise find ZAMC. Solution: 2A = 80°. AN is angular bisector of ZA. (in AABC) => ABC = ZACB = 50°. But ZCBM = 30° = (Gama) Also given ZBCM = 10° - Consider AABN and AACN. a AB AC (given). AN is common ZBAN CAN (given) Ul] <. By SAS congruency, * © AABN = AACN => [BN =CN | = | ABNC is Isosceles «. 4BCN = ZNBC = 30° > (OW = 86Problems and Solutions 87 We have 4ANB = ZANC = ZBNC = 120° Consider AANC and AMCN., They are congruent by ASA property. » AN=NM, (corresponding sides) Hence ZNAM = ZNMA 180° — 120° = BOS a0. -. ZAMC = ZAMN + ZNMC = 30° + 40° = 70° 2. Semicircles are drawn externally on the sides of a given triangle ABC. The lengths of the common tangents of semicircles on AB and CA is p, on AB and BC is q and on BC and CA is r. Then show that semiperimeter of the given triangle is 24042, Solution:. Let AB = c, BOC =a, CA =b and let A’, B’,C’ be midpoints of sides BC,CA,AB res- A pectively. : Let the common tangent Sy to semicircles on AB, . Nae BC meet them at the + points of contacts K,L respectively. The radii of semicircles AB, BC, A ‘are §,9,$ respectively. C'A = }CAr§.88 Problems and Solutions Draw quadrilateral O’A'LK. C'K 1 KL, AL 1 LK (tangent is perpendicular to the radius at the point of contact). Draw C’M || KL. We get aright AC’A'M. By Pythagoras theorem, ($)” ~ ($~ §)” = 4? bta-c b+c-a aece uaa a) ‘ce +b-c ate Similarly p? = S75 — 2, 24678 (2) ¢ rp cta-b ctb-a Likewise, | fie PF Ee 8 Pp 2 *, B+ E42 =} (a+b+c) = semi perimeter of AABC. 3. A 10 digit natural number with no repeated digits and leading digit non zero is called a magic number if it is divisible by 99999. How many such magic numbers are there?Problems and Solutions 8g Solution: Let N represent the magic number end be given by NN = agaga7aga50403020109 where a; represents the 10‘ th place digit of N, 0
[aodaardgas’] + [eaageadidy | = 99999 as all digits are different which means the addition involves no carry over. .". Independently, we have a9 ag ay 06 a5 +a, ‘+03 to. +01 +00 9 9 9 9 9 Number of ways of choosing a9'= 9 80 as (ag, as) Number of ways of choosing a3 = § 50 a3 (ag, a3) Number of ways of choosing ay = 6 so as (a7,a2) Number of ways of choosing ap = 4 80 as (a6, 41)90 Problems and Solutions Number of ways of choosing as = 2 80 as (a5, 49) By Fundamental Principle, Number of magic numbers = 9 x 8 x 6 x 4x 2 = | 3456 4. Prove that 1.14+1.2.2+41.2.3.34+-°+12--(n-1)-n-n= L2-en(ntY—1. Solution: 1-14 1-2-241-2-3-34-:-+1-2-3-+-(n—1)-nen = 1-(2-1)41-2-(3-1) $1-2-3- (4-1) +e +1:2+3-.-(n—1)nf +1 - I] = -LL41-2-1-241-2-3-1-2-3 H1-2:3-4-1-2-3-4406 41-2-3---n—1-2-3---n$1-2-3---n(n +.) (Telescopic cancellation) = -14+1-2-3-.-n(n+1). Hence proved Nate: The result can be also obtained by mathematical induction. 5. Suppose that a,b,c are positive real numbers. Prove thatProblems and Solutions 91 (-4) +G-4+G-3)| o Gtbepeegiay 27 Leger)? (byte : S (4b) (EEE #) 2 0) Consider 3, 3,2. Applying A.M-G.M inequality to the positive reals, we get Ayiyl eters > (att 1 2 = (HE) > (abe) (2) Consider %,%,. Applying A.M - G.M. inequality to the positive reals, we get ab 4 bs Etat 2 (aio eo 2 > (434) > (abe)$ (3) Lgdgr)\? (yey ey? (2) x 3) > (4 + \. (238) 21 which ii (1). Hence proved.92 Problems and Solutions 6. Find all triples (x,y,z) of real numbers satisfying the following three equations (where [r] and {r} denote the greatest integer
|{y}+ [z] = 99.95 atyt+z—({z}+y+[2]) 100.05 => ‘[[c} + {2} = 100.05 ttytz—([z]+{y}+2) " 100.15 => | {x} + [y] = 100.15 fu}+le] = 9995 > [7] = 99, {y} = 0.95 le]+{z} = 100.05 = [2] = 100, {2} 0.05 {z}+[y] = 10015 = [fy] = 100, {2} = o15 . @ = {2]+{2} = 100.15 vy = (y+{y} = 100.95 z = J+{z} = 99.05 7. Determine all distinct triangles having one side of length 6, with the other two sides being integers, and the perimeter numerically equal to the area. Solution: Let a = 6. Without loss of generality 6 < ¢ where a,b,c are the side lengths of tho triangle required. From given information, Vals = a)(s—B)(e—c) = 26 where s is semi perimeter of theProblems and Solutions 95 triangle. => (s—a)(s—8)(s—c)=48 : => (b+c-a)(c+a—b)(a+b—c) = 16(a+b+c) => (b+c—6)[36 — (c—b)?] = 16[6+e—6 + 12] => (b+e-6)(20-(c—b)?)=192 where b+ >6. *. %—(c-b)?>1 as cE ZS0Sc-b<4. But 20—(c—b)? should be a factor of 192. v. (= 8) can be 4 or 16 only. (ct =4b4e-6= = 2b tem 18 (or) c—b=2, b+c= 18> [¢,b= 10,8 If (c- 6)? =16, 6+c-6= 12 = 48 > b+c=54 (or) cb =4, b+ c= 84> [eb — 20,95 ~. There are 2 triangles with its perimeter numerically equal to its area (one of the side length is 6 units) They have the side lengths. es (6,8,10), (6,25,29). 8. A Swiss bank cellar is protected by 7 security men. The bank cellar has 10 doors and every door has 3 locks. All locks are different from one another. Bach security person has the keys to some locks so that any four security persons have keys to all locks, so that they can open the cellar in an emergency. Prove that there exist three security persons who together have keys for all locks. Solution: There are 7Cs = 35 sets of 3-security men. Let us assume that no such set of S-security men have the roquired set of all keys which will open all the 10 cellar doors. ‘There are 30 keys and 35 sets. By pigeon hole principle, at least 2 sets of 3-security people must not have the same key say94 Problems and Solutions “K’, As these 2 sets are distinct, they must differ in at least.1 person; For example if one set .A = {a,b,c} the other set may be B = {a,b,c,d} (where a,b,c.d are some 4 security men) differing in atleast 1 person. Hence we have the set {a,b, c,d} having 4 elements not having thé key ‘KX’ which contradicts the given condition. Hence there is a set of 3 people which has all the keys to all the 10 doors. 9. Let Rn demote the remainder obtained when 3%” is divided by 2"+3 -Find Ragor - Solution: It can be verified that R; =9, Ra =17 1 gat ie., remainders of 33, Sry respectively. We can write Ry = 1+2)4?7 = 9 and Ry= 1+ 27+? = 17 suggesting thet 2, can be 14+ 2"+?, Assume that Ry, =1+2"?, If the quotient obtained when 3”” is divided by 243 is Qn, then 37 =1+2°74Q,-2"8 (By Buclid's Algorithm) gmt Bey = 14.2007 4.Q,. 2048) +(2t3 + Qn os gnts)2 1+2"5(mod 2"4] i Thus Rag, = 1+ 2943 = 1 4 glnthyt2 Hence by induction, Ry = 1+ 2"+? for all ne N. wo Rawr = PPR].BHASKARA CONTEST SCREENING TEST - 2008 1, Two semi circles are constructed as shown in the figure. The chord PQ of P the greater circle touches the smaller circle and is parallel to the diameter of larger circle. If the length of PQ is 10 units, then what is the area between the semi circles? (A) 50 (8) 507 (C) 4.257 (D) 3.25 2 Solution: Let the radius of the bigger circle be R units and the radius of the smaller ot circle be 7 units and let ‘0? axeca be centre of the bigger circle. ! PQ =10 unite and PQ is parallel to the diameter + Ra-pP = 0Q?-? = MQ? (M is the mid point of PQ) 10\? _ (3) = 25. Area, between the semicircles is }[(R? — r2)] i = } Xx 25 oq. units Answer (C). 9596 Problems and Solutions 2. The digits 1,2,3,4,5 and 6 Ta] T fat im are written in a spiral like [ria ys [5s ca) z fashion beginning from e{3t4isieli 213) the certtral marked cell. s[2{tl213 [413 Whi digit is written on [PETTERS TS TS the cell placed exactly 100 stetst2 tts cells above the marked 1 erst 14 cell? a] (A)1 @)3 (C)5 (0) 6 Solution: 100 cells above the marked cell is contained in the square with (2 x 100 + 1)? = 40401 cells totally (how?) The right top comer is the 40401th cell but the numbers are written from’ to 6 respectively, this right top corner will be the umber [40401 mod 6 = 3(mod6)]. The middle of the strip of this largest square is 100 cells to the left of it or 100 (mod 6) from it = 4(mod6) from it, So thin number is 3~ 4 = —1(mod 6) or —1+1=5(mod6). So the 100th cell above the marked cell has the number 5 in it. Answer (C). 3. An island is inhabited by both liars and knights. Every knight always tells the truth and each liar always lies. One day 10 islanders gathered together and issued a few statements. Two. people said “Exactly two of us are liars”. Another four peogle said “Exactly four of us are liars": The remaining six said “Exactly six of us are liars". How many liars are there? (A) 10 (B)8 (o)6 (D)4 Solution: In a group since all are making identical statements either it is true or false. If true all in the group are knights; if false all are liars. So if the first two and the next four lie it means the last six are making true statements. Hence six are liars. Answer (C).Problems and Solutions 97 4, For how many real values of a will 2? + 2an + 2008 = 0 has two integer roots? (a2 (By 6 (p) 8 Solution: Let a, 8 be the two integer roots of 2? + 2ar +2008 =0. Then «+6 =—2a, af = 2008. 2008 can be written as the product of 2 integers in the following 8 different ways. (1) 1x 2008 (2) 2x 1004 (3) -2x -1004 (4) -1 x -2008 (5) 4x 502 (6) ~4x —502 (7) 8x 251 (8) -8x -251 (as a and f are integers) o + 6 = —2a is also an integer. Thus «+9 can be odd or even since a is real. Again 251 is prime and so the above are the only possible factorizations of 2008. . a= HA) will have 8 values. Answer (D). 5. The first digit from the left of a 4 digit number is equal to the number of zeroes in the number. The’second digit is equal to the number of digits 1, the.third digit is equal to the number of digits 2 and the fourth digit is equal to the number of digits 3. How many numbers have this property? (A) oO (B) 2 (C) 3 (D)4 Solution: Let the four digit number be 4yA,AgA3 where A; js the number of 1's in the four digit number Ag # 0 being the98 Problems and Solutions > leading digit Ao cannot be 3.as As must bevatleast 1 Ay = 2, means there must be 2 zeroes and A; > 1. Ag must be zero and A; cannot be > 1. In thia case the only solution is 2020. Again 49 =1. Ay is atleast 1. Either Az or Ag must be zero. If Az =0, As 21 and hence 3 must occur among Ap and A which is impossible As=0, A, =2 ie, 1210. Thus the number of solutions is 2. Answer (B). Let A be the least number such that 10A is a perfect square and 35A js perfect cube. Thon the number of positive divisors of A is (A) 72 (B) 64 (©) 45 (D) 80 Solution: 10A =-2 5x A is a perfect square. (ie.) In the prime power representation of 2 x 5 x A, the powers are _ even. (i.e) A must have odd power of 2 and odd power of 5 as factors, the remaining factors, for 10A to be a perfect square. ie, A= 22e+! x 527+). 2, where x is a perfect square. In prime factorization of 354, the powers-are multiples of 3. In A, the power of 7 is 3k—1, the power of is 31—1-and the rest of the factor is a perfect cube. A = 7&1 x 5-1. y where y is 8 perfect cube. Thus god 52641. = 7Sk-L 58-1 oy .. 7 must divide z and 2 must divide y; 2 must be a perfect square and y; a perfect cube. ‘The least values of 28-+1 = 31~1 is § and the least value of 3k i-1= 2 and the least value of 2a+1=3. 2 AaB. 2, ., Number of factors of A is (3+1) x (5+1) x (241) = 72. Answer (A). .Problems and Solutions 99 7. The number of positive integer solutions of ab~ 24 = 2a is? (Ayo (B) 2 (C) 8 (D) 12 Solution: ab— 24 = 2a => a(b— 2) = 24 and b> 2 = a(b—2)=2% = 1x 2M=2x12=3x8=4x6=6x4 = 8x3=12x2=%x1 => (a,b) = (1,26), (2,14), (3, 10), (4,8), (6,6), (8: 5), (12, 4), (24,3) ‘The number of positive integer solution is 8. Answer (C).. 8 What is the least possible value of the expression 2008 ~ BHA~SK—ARA if it ia known that each alphabet represents a different non zero digit? (A) 106. (B)108 © (C) 1580 -~—(D) none of these Solution: 2008 — BHA — SK ~ ARA has the least possible value when. BHA + SK +ARA hes the maximum value. For the 100! place digits must be 9 and 8. Since A is repeating again A=9, B= 8. 10* place digits must be next higher digits and hence H,S and R must be 7,6 and 5..K must be 4. v. BHA+ SK + ARA = 879 + 64 + 959 = 1902 and 2008 - BHA — SK ~ ARA = 2008 — 1902 = 106. Answer (A). 9, In one move the king can go to any adjacent square, along a row, column or diegonal. How many routes with the minimum number of moves are there for the king to [—]~} travel from the top left square to the — bottom right square? (A)35 (B)2 (C)15 (D)4100 Problems and Solutions Solution: There are 4 rows and 5 columns in- the board, ‘Taking the diagonal step king moves down as well as right. He needs 4 diagonal steps. Steps are 1 right 3 diagonal as shown below: KG oP : : ie G) > (i X cE K Ky kK ace Wo ot we, ek, ‘Thus there are four ways with 4 moves and 4 moves isthe least Answer (D). 10. pei o win t+ wis t asi te + 25s 24/25, = (A) f (8) ¢ © # © % Solution: ‘The given expression 1 1 1 peuaeet Seaeeeen Papo ts ON QWi+1v2 3V2+2V8 . 25V24 + 24/25" can be found to be ed 1 Letiaiaeti 2 (n+ 1a 2+ (n+ i)Pn—w(n +1) a - Some Dvi=n mt ‘a n+1)n bal - LGe- em)Problems and Solutions 101 = AeA ta tet Viva a Va a VB = 4-12-12! vil ¥% 5 5 Answer (B). 1. The number of pairs of prime numbers (p,q) satisfying the condition {Jy <}+1<§ will be (A) 49 (B) 24 (C) 50 (D) 48 Solution: Given $< 1+ <§ where p,q are primes. For primes p> 5, q25, we have 1,1_2_ 40 51 1 =<: ‘ + 95575 ~5 = i00 < i00 : Pp "Thus p and q cannot be simultaneously greater than or equal to 5. One of them must be 2 or 3 only. Suppose g = 2. a 51 Ae woo
Valeva + yy) = 336 and Joly + zVz) = 112. ve _ 336 = Veo a 3 = 8. from 27+ y Jay = 336, we get aly +y- VOR = 336 or Bly? +3y7 = 336 or Sy? = 336 o yt = 4, y=2. Hence s+ y = 10y=10x 2 = 20. Answer (C).Problems and Solutions 107 20. ABC is an equilateral Q. G is the centroid aud AG = 2em Ss is the circumtircle of the triangle. The area of the shaded portion (in cm?) is (A) (4m -3V3) (B) iad (C) 3(4m — 8v3) (D) }(4x - 3V3) Solution: AABC is equilateral AG = 2cm=> AD = 83cm and the area of the shaded portion is the area of circle minus the area of equilateral A Radius of the circum circle 5 is AG=2. Side of the equilateral A ABC is 2x3 = — =2¥3 om avs . Shaded erea = 4r — x 4x 3. = (4x — 3V3) sq. cn. Answer (A). 21, The number of integer solutions of the equation F(4—2) = +4 is (A)3 (B) 4 (c)o0 (D) infinite Solution: B4—2) = +4 ae = 2et4 4-2108 Problems and Solutions za0e1 = f=1 z=132 = Pit a2 extad = “thay and for negative integers and for positive integers greater than 2, we do not have both sides of the equation satisfied. ‘Thus the equation has 3 integer solutions. Answer (A). [The ‘given equation is equivalent to 2? = 2444; for 2 = 3, we get 8 = 10 a contradiction 2 =4 undefined and for «> 4 2 iy a negative number, again a contradiction. If z is negative say x= —a where a is an integer > 0, then 27 is a positive unit frection with the denominator a power of 2. ie, 27% = 4222, clearly 4— 2a is less than 4+a for a= +1, we get 442= 3, c= 2, 9, 223 weget $522 is a negative fraction. ‘Thus the only possible integer solutions for x are 0,1 and 2]. 22. If a,b,c are positive integers such that a? + 2b? — 2ab = 169 and 2be—c? =169 then a+b+e is (A) o. (B) 169 (©) 13 (D) 39 Solution: o? +26? — 2ab = 169 and 2bc — c? = 169 => a? +26? — 2ab- 2be+c? = 0 = (a-d+(b-o? = 0 => a=bb=cProblems and Solutions 109 2. 2be= 2 = 169 sc = 169 (c is a positive integer). 1 @sb=c=l3 “O+d+0=3%x 13 = 39. Answer (D). 23. Two of the altitudes of a scalene triangle have lengths 4 and 12, Tf the length of the remaining altitude is also an integer, then its maximum value is (Ay (B) 2 (o)5 (D) 8 Solution: Area of the AABG is tac = } x A 4a = $12 x b. = cr 4a 12 ip, < aoa — ¢ anda = 3b. The ., AC+BC>AB>b+a>c34b>c (1) and BO-AC
2%
lbh = V48 x 18 X24 = 24 x 6 = 144 cubic units lbh _ 144 t= Hh = yg = Som pe cm. -th 24 ho Oh Ma ib 48 t+b+h=8+8+3=17em, Answer (A). 25. ABCD is asquare. With centres B and C and radius equal to the side of the square, circles are drawn to eut one another at E inside the square. ZBDE is equal to (A) 22}° (B) 30° (C) 15° () 37° Solution: The radii of the circles with caiitres B and C are each =a, the side of the square. a B D Te, ABEC is equilateral.Problems and Solutions 111 .. LBCR = £CBE = 60° ZECD = LEBA = 90° — 60° = 30° CE=CD = BE= BA -. SAEB® ADEC ZBDG = £DEO = /BAB = BEA = 2O=9) «298 BDC = 9 = 45° ZEDB = ZEDC — ZBDC = 75 — 45 = 30° = (4BDE) Answer (B). 26. p and q are primes. It is given that the quadratic equation x? — pz +q=0 has distinct real roots. Then (p+ 2q) is (A)7 (B) 19 (c)9 @) 18 Solution: If a and § are the distinct real roots of at —pe+q=0 where p,q are primes, p* — 4g > 0. ‘As af = q, @ prime number, we have a = 1, 8 = q oF a= q, 6=1. Hence a+f=p implies g=p—1. As bath qand p are primes, p=3, @=2. Hence p+2g=3+4=7. Answer (A). 27. The number of two-digit numbers that-increase by 75% when. their digits are reversed is (A) 3 (B) 4 (05 (D)7 Solution: Let the numbers be 10a+b-and 105+ and the reversed number 10b+4 is 75% moré than l0a+. ie, 10b+a= J x (10a +b) => 40b+ 4a a+ 7b => 33b 66a => b = 2a. since a,b are the digits, the only possible values of a and 6 are (1,2), (2,4), (3,6) and (4,8) so the numbers are 12, 24, 36-112 Problems and Solutions and 48, the numbers with reversed digits being 21, 42, 63 and 84 and there are just these 4 numbers. Answer (B). 28. Arun thought of a positive integer. Balu multiplied this by either 5 or 6. Chitra added 5 or 6 to Balu’s number. Devi subtracted either 5 or 6 from the number got by Chitra and the final result was 73. Then the numbers obtained by Arun, Balu and Chitra are respectively. (A) 12, 72,78 (B) 12,60,66 (CG) 12,72,77 (D) 12,60, 65 Solution: Let us follow the sequence.of numbers written by Arun, Balu, Chithra and Devi as follows Arun Balu Chithra Devi . a Sacr6a Sa+5ora+6 §0,50—1,5a+1,5a or6a+5or6a+6 6a,6e—1,6a+1,6a The final answer got by Devi is 73 ie, 5a=78, Setl=73 Sal = 73 6a=73, Ge+1=73 6x1 =73 Of these only 6a +1 = 73 gives an integer value to a ie. a=12. Arun wrote 12; Balu should’ have written-72 Chithra should have written 6a + 6 = 78. Answer (A). 29. AB isa fixed diameter of acircle. PQ is achord whose length is equal to the radius of the circle, which docs not intersect the diameter AB.. AP and BQ meet at R (A) ZARB = 45° when PQ is parallel to AB. (B) ZARB = 60° for all positions of PQ (©) ZARB = 60° only when PQ is parallel to AB (D) ZARB is 6 variable angle depending on the position of PQ. :Problems and Solutions 113 Solution: In the diagram OP =0Q = PQ=OB=OA R a, 2P0Q =60°; ZPAO = ZAPO == (say) ZQBO = £BQO = y, say ‘Then ZQOB = (180 ~ 2y) and ZPOA = (180 — 22) 5. 60° = ZPOQ = 180° [(180 — 22) + (180 — 2y)] = %z+y)—180 240 orsty = = 10° 2 . ZARB = 180-[4RAB+ZRBA] = 180°-(z+y) = 180-120-607. Thus ZARB = 60°, irrespective of the position of P and Q. Answer (B). 30. a,b are positive integers If logy.a = logy, = logys(a +b) then the value of (2) is (4) ¢ (a) 48 8 (D) 436 Solution: If logy a = logya b = logyg(a+) =k then a=9',b=12* and a+b=16"114 Problems and Solutions +, 97% 4.2% x gk 94. Let 3* be 2 oP 4 Wy — otk = 9 +g? 4 gthg — fk as x = 3* can not be negative. Answer (D) Now oz = 0 2 = 22h VEBHASKARA CONTEST FINAL TEST - 2008 1, The number of students in a university is 8 perfect square. In one year 2000 more students joined the original strength and the new strength is one more than a perfect square. The next year 2000 more students joined newly and the new strength is again a perfect square. What was the original strength of the university? Solution: Let -2? be the original strength of the University. After 2000 students joined, let the strength be y?+1. In the subsequent year, after 2000 more stydents joined, let the strength be z?. We have 7 +2000 = y? +1 implying y? — c? = 1999 Q) and y?+1+ 2000 = 2? implying 22—y?= 2001 (2) (1) + (y—2)(y+2) = 1999, which is a prime number. Hence y~z=1 and y+2= 1999 . 2y = 2000 or y= 1000 and z = 999. From (2) 2 — (1000)? = 2001 or 2? = 10007 +2 x 1000+ 1= (1001)? “2 = 1001. ‘The original strength= 2? = (999)? = (1000 — 1)? 1000000'— 2 x 1000 +1 1000001 — 2000 = 998001 115116 Problems and Solutions » . In the adjoining figure ABCD is wrectangle, Triangle PAB is isosceles. The radius of each of the smaller circles is 3cm and the radius of the bigger circle is 4cm. Find the length and breadth of the rectangle. Solution: a In the figure, let AD =b and DP =a. PS = PQ~SQ=b-+ SY =4, AX = AM =b-3 PL = PX =a-3 and AY =a PY = PX - XY = PX - (AY — AX) = PX -AY 4+ AX -3-a+b-3=6-6 Using Pythagoras Theorem, from right angled APSY , we get (b= 4)? = (6-6)? +4? vB? 8b +16 =o? + 36 — 126+ 16 or b=9 Hence PY =b-6=9-—6=3 and tend = $% = 4. From AAQP, tand = $3 = $= §.Problems and Solutions 117 naa $x9sl2. Length of the rectangle = 2a =2x 12=24. Breadth of the rectangle =b=9. 3. For a natural number A, denote the product of the digits of A (in decimal base) by P(A). Determine all values of A such that A= 1.5P(A). Solution: If A is a single digit number, say A = a we have a=15e>a=0. ‘Thus the number of digits of A, say n is greater than or equal to2. (n>2). Let A be en‘n’ digit number with leading digit m. ‘Then, clearly A>m-10"" q) If every other digit of A is 9, then P(A) =m-9"-1. If A=1.5P(A), m1 < mgr s1t< 3 sgn (2) Also, since 9/P(A), 9/A=> m=9 But this means 1.5P(A) is not an integer. So, P(A)
12 x 81 = 972. ‘Thus we have to look for two and three digit numbers only. Let A be such a two digited number ab so that A= 100+5. Since A= $P(A), we have lat b=§xaxb :.a(1.5)—10) =6. As b= RHS is an integer, 1.56 is also an integer. je } is an even integer. Also 1.5b— 10 is positive, Hence 6> 3§ = 6.66. “b=8 or9. As b is even it can be only 8. Hence a(1.55—10) = 6 gives a = 4, Thus 48 is the only possible two digited value for A. Next, when A is a three digited number abe, we have 1000+ Wb+c=15xaxbxe 10b + ¢ = (1.5be— 100)a. (5) As 1.5be — 100 > 0, we must have be > 66.66. .-, bc > 67 and must be even ( bc is an integer). Hence b = 8,c = 9 or b=9,c=9. Both these do not satisfy (5), Therefore, A is not equal to any 3-digit number. Thus A has only 2 values 0 and 48. 4. A rectangular cuboid has integer dimensions. All of its faces are painted green, The cuboid is partitioned into unit cubes by planes parallel to its faces. Find all possible messurements of the cuboid, if the number of cubes, without green face is one-third of the total number of faces. Solution: Let the cuboid’s dimensions be a,b,c. These Jengths must all be at least 3 or else every cube has a gresD face. The given condition is equivalent to 3(a—2)(b—2)(c—2) = If all the dimensions are at least 7, thenProblems and Solutions 119 a b 7\8 _ 343 : g22 b-2 xas(%) Fag <3, scmtmadiodon. ‘Then one of the dimensions say ‘a’ = 3,4,5 or 6. ‘Without loss of generality, let us assume b < c. When a= 3, we have be = (b—2)(c—2), which is impossible. ‘When a = 4, rearranging the equation, we get (= 6)(c- 6) = 24 = (be) = (7,30), (8,28), (9,14) oF (10,12). When a=5, we get (2b—9)(2c—9) = 45 = (bye) = (5,27), (6,12) or (7,9). When a= 6, we get (b— 4)(c- 4) =8 => (b,c) = (5,12) or (8,8). Therefore, the cuboid may measure 4 x 7 x 30. 4x 8 x 18, 4x9x14,4x10x 12, 5x5 x 27, 5x6x 12, 5x7x®9, 6x5x12 or 6x6x8. * ABCD is 0 quadrilateral such that AB = 2% ry +12), BC = 2(ra +173); CD = 2A(r3 +74) DA=2%Xr4+r1), AC = 11 +205 +13, BD = rat 2rst+ra where T1,asT3,Tay 75 ZO Prove that (r1 +172 +53 +04 — 4rs)(rirargra — 7) > 0 Solution: Let O be the Point Of intersection of the diagonals AC and BD. By triangle inequslity, we get AO+OB > AB BO+O0G > BC CO+OD > CD DO+OA > DA120 Problems and Solutions Adding we get 2(AC + BD) > AB+BO+CD+DA (Ql) Given: AB = Ary+r2) BC = %Arm+rs) CD = %rs+r4) DA = 2ra+r1) AC rit 2r5 +73 BD = r+2rg+r4 andry,rats,rs 20 () ri tretratra <4r5 (2) By AM-GM inequality we get : ritratrs +r : > Yann strstr o, From (2) r5> DAR ERIN > yar ot rararara < rf = (rararara — rf) <0 (3) From (2), (3) we get (ri + 72+73 +14 —475)(rirarara — rf) > 0 6. Let z,y,2 be three positive reals, each lesg has 4, Prove thet atleast one of the numbers 3+ 745,24 71,14 ,1- is greater than or equal to 1. Solution: From AM-HM inequality we have if, 4 QM phen) 3(2 7 3)2 zy4-a 2 ia =+7—> Stace?!121 Problems and Solutions Similasly, b+ >a, 4 If the three numbers i+ i Tess than 1, their sum S$ would be less than 3. But » d+ ay. b+ qk were all <. At least one of the numbers is greater than or equal to 1. 2 ‘The sum of two adjacent angles of a trapezium is 90°. The lengths of the two parallel sides are ‘a’ and ‘b’ respectively. : Then prove that the length of the line segnient joining the mid- points of the two parallel sides is $a — 0). Solution: a ¢ A\ 4 Papeete Drew CM DL perpendicular to AB. (Assume a > b) Q,R are the mid points of CD and AB respectively. Let ZA=@ and 2B=90~8. Clearly DL =CM =h (Say) tand.= 9b = fy => h= ALtan@ = AL = heote tan(90 — 9) = Gif = giz > BM = Aten?122 Problems and Solutions Now, AL+LM+MB heoté+6+htand b+ A(tand + cot 0) 1a ~b= h(tand + cot é) Q) PR=LR-LP= AR~AL— LP =§~—heotd—} = 95¢ — hoot # = S(tand +cot9)—heot@ (from(1)) “. PR= }(tari@— cot 6) Now, PQ=h, PR= h(tend —cotd) ‘Therefore, from right angled APQR, "QR PQ? + PR? = 1+ 2 (ean ~ cot) AB ui re = F ((tene — cot)? + 4) 2 = © ((and - cote)? + 4tand cot) 2 = F (tane+ cot 8)? ie, QR = Kana + cotd) 1 7 = 7(2—0) (using (1) Alternate Solution: Tet ABCD be the trapezium in which AB = a, CD (Here we have assumed that a>).Problems and Solutions 123 Let P,Q be the mid points of AB, CD respectively. Through C draw a line parallel to AD to meet AB at P and a line parallel to QR to meet AB at S. Given: 2A + ZB =90°. Since APCD is a parallelogram, ZOPB = ZDAB. implies ZPCB = 90°. To prove: QR = }ia — d| Since QRSC is a paralleleogram, we have QR = SC. -. We have to prova SC = }Ja— Now, BS = BR-SR=}AB~QO~ (AB -CD) ie, BS = }o-8) PS =PB-B9= AB- AP - BS =a—b— (252) = 938 So, $ is the midpoint of PB. Since APCB is right angled, S$ is the midpoint of the hypotenuse, and QR=CS = BS = PS = }(a—-b). 8. Let a,b,c,d, Pe pose 64 Prove that 5454549? apbeore Solution: We have ta prove that ie,124 Problems and Solutions 16b LHS = 1+ oe +e bi ® he d £84. fet +s tate a c fod n+ (t+ 2+ (2 8) ($248) d 2b 4d +0(F+ £)+2(2+ +g) (F+5 Since a,b, ¢,d are positive reals, 2a, c da, d 2 Ma cy, 4a dy, & 2B Stee Gta? 7 ore ais LHS’ > 22424 2(2) +4(2) + 8(2) + 2(2) + 4(2) = W4+24+44+84+164448 > LHS > 64 9. ABCD and AB‘C'D' are two i‘ R squares arranged os in the figure. DD! and BB intersect at P. Z Prove that (a) AP is perpendicular to CC’. > (b) Find the ratio of BB! to CC’. - Solution: P is the point of intersection of BB! and DD'. Join A and P. Consider the triangles ADDY and ABB! AB! = AD! (sides of the square AB'C'D') AB = AD (Sides of tbe square ABCD) ZDAD! = 90+ DAB ZBAB' = 90+ LDAB'. :, The triangles are congruent. -, ZABB! = ZADD! => B,A,P,D are concyclic.Problems and Solutions 125 But the circle passing through A,B, D passes through C’. :. A,B,C, D,P are coneyclic. Similarly A, D/,0',B', P are also concyclic. ZAPC = 90° (angle in the semicircle with AC as diameter) ZAPC! = 90° (angle in the semicircle with AG’ as diameter) OPO? is a straight line and AP is perpendicular to CC’. Let AB = a, AD’ = b. Consider the three triangles, ABB’, AADD! and AACC'. > ; : Y : 9 Clearly AABB! is congruent to ADD", and both are similar to AACC’ +. Since AC = V3a, AC’ = V2 we have CC! = ./2BB' BBL “OO” a BB: OC! =1: V2BHASKARA CONTEST SCREENING TEST - 2009 1. Six equal circles are arranged as in the figure. The height of the triangle is 4. Then the radius of the circle is W-1 @ ( (C) Wa (D) v3 Solution: Let r be the radius of the circles. Look at the rhombus PQRS. v3-1 2 PQ = 2r, PR = 2PQcos30° = 2- height =4 = 2V8r + Or = 2(V8-+1) +4 oe wn) Answer (A). 2. The number of squares on a coordinate plane with one vertex at A(—2,2) and atleast one of the coordinate axes as axis of symmetry of the square is (A) 3 (B) 5 (C)6 (D)7 Solution: There are 5 such squares. See Question 2 of Inter. Answer (B). 3.The number of real solutions of the equation (z+1)(3*—2) =1 is (A) 0 (B)1 (C) 2 (D) more than 2 126Problems and Solutions 127 Solution: (e+1)(3 -2)=1 1 37 -2=—_ a+1 Number of solutions of (« + 1)(3* — 2) = 1 equals Number of 1 points of intersections of y=3"—2 and y= —1— z+1 1 For co>z>-l,ie,co>t+1>0, coo>——>0 a+1 1 —oo
0. If m is triangular, then (8m + 1) is (A) also triangular (B) a perfect square (C) a perfect cube (D) none of the above three Solution: nerd ie, IM=n24n 2.8m = 4n?+44n=(2n)?+4n Clearly 8m is not triangular. But 8m +1 = (2n+1)? isa perfect square. Answer (B). 6. Let f(n) be the number of non-congruent integer sided triangles with perimeter n (f(3)=1, f(4)=0, f(5) =1). Then f(10) is (A) 2 (B) 3 (c)4 (D) greater than 4 Solution: For perimeter to be 7, no side of the triangle can be >. Allsides are < }. For n = 10, the longest side is 4. If the longest side is <3, then perimeter cannot be 10. . One side must be 4. . Only two such triangles exist. (3,3,4) (2,4, 4) Answer (A).Probleme and Solutions 129 7. Ananya wrote all the integers of 1 to 6 digits. She uses only the two digits 0 and 5. In all, how many 0's did she use? (leading zeros not allowed in the numbers) : (A) 256 (B) 128 (C) 129 (D) 200 Solution: No. of 1 digit numbers 2: No. of zeroes in it 1 No. of 2 digit numbers 2: No. of zeroes in it 1 No. of 3 digit numbers 4: No. of zeroes in it 4 = 442 No. of 4 digit numbers 28 = : : No. of zeroes in it 853 = 12. 6: x4 39 2x5 _ 99, No. of 5 digit numbers 24 = 16: No. of zeroes in it 4 No.of 6 digit numbers 2° = 32: No.of zeroes in it Total zeros used 130. If 1 digit number zero is ignored, total zeros is 129. Answer (C). 8. The diagonal BD of a quadrilateral ABCD bisects the angle ZABC. It is given-that AC = BC,ZBDC = g0° ZACB = 20°. ZBAD is (A) 110° (B) 120° (C) 130° (D) 140° B Solution: ZBAC = ZABC 180-20. = >=
16. The number of 100 digit numbers having the sum of their digits equal to 3 is Solution: 3 will occur as sum in the following 3 ways 3,2+ 1,1+1+41. With a single 3, there is only one number 3 with 99 zeros with a2 and 1, ai: 201 20---1 3 = 1+1+1 In the last 99 digits 2 ones will occur in 99C2 ways 99 x 100 (ie) 5 = 4950. Total number of such numbers = 4950+ 99 £ 99 +1 =5050 +99 =5149.Problems and Solutions 135 17. The value of 1+ 2008/1 + 2009/1 + 2010/1 + 2011 is ——— Solution: 1+ 2011-2013 (2012 — 1)(2012 + 1) +1 = (2012)? vI+2010-2012 = 2011 vi + 2009-2017 2010 v1+ 2008-2010 = 2009 Answer is 2009. 18. Each term of a sequence is the sum of its preceding two terms from the third term onwards. The second term of the sequence is —1 and the 10th tern is 29. The first term is —————— Solution: Terms are a,b,a+b,a + 2b, 2a + 3b,3a + 5b, 5a + 8b, 8a + 13b, 13a + 21b, 21a + 34b b=-1 21a + 34b = 29 21a = 29+34=63 |a=3 19. The number of positive integers satisfying the inequality 2000 < /n(n + 2) < 2009 is Solution: 20002 < n(n+2)< (2009)? Add 1 2000? +i < (n+1)?< (2009)? +1 2000? +1 < 2001? < (n+ 1)? < (2009)? < (2009)? +1 2001 < n+1< 2009 2000 < n< 2008 The number of positive integers is 9.136 Problems and Solutions 20. The number of pairs of positive integers (z,y) satisfying the equation x?-+y? + 2ry —2008x — 2008y— 2009 = 0 is ——__ Solution: a? + y? + 2ry — 20082 — 2008y — 2009 (x+y)? — 2008(x + y) — 2009 (z+y—2009)(2@+y+1) 0 oo z+y = 2009 orz+y = —1 2+y = —1 does not cross I quadrant. 2 +y = 2009 (0,2009),(2009,0) in I quadrant. Both (x,y) positive are from (1,2008) to (2008,1). So there are 2008 such numbers. 21.Given a,b,c > 0 and a+b+c=1 then the maximum value of Yatb+ Yo+e+ Yera is ——— Solution: a,bc>0 atb+e=1 Va+b+ Vo+c+ Yeta=Vi—c+ Yi—at Vi-b a,b,c > 0. This is clearly <3. But it will not equal 3, since in that case a=b=c=0 violating a+b+c=1 Ibis <3. ‘The value by symmetry of the expression is 22. All the numbers from 1 to 2009 are written one after the other. ‘The number of digits in this number is Solution: No. of 1 digit numbers 1 to 9 — 9 No. of 2 digit numbers 10 to 99 — 90 No. of 3 digit numbers 100 to 999 — 900 No. of 4 digit numbers 1000 to 2009 — 1010 ‘Total number of digits is 9+ 2x 90+3 x 900+4 x 1010 = 6929 If 2009 is not included then no. of digits is 6929.Problems and Solutions 137 23.A square is described on the hypotenuse of a right angled triangle away from the right angled vertex. The ratio of the perpendiculars drawn from the intersection point of the diagonals of this square to the legs of the right angled triangle is Solution: OC OA ZOCM = ZACB+45° 0+ 45° ZNAO = 180~ZOAB 180 — 45 — (90 — 6) 4540 - MONA ASOMC (since all 3 angles are equal and hypotenuses are equal). “ON =OM o. Ratio is 1. 24. PMT isa tangent to the circle APC at the point P; CNAT is a diameter, to which PN is drawn perpendicular and AM is perpendicular to PT. The ratio of AM to AN is —— Solution: O centre of the circle OPLPMT ¢ Let ZAPM =<. Then ZAPM = ZPCA® (angle in alt. segment) CA is diameter.138 Problems and Solutions ZCPA = 90° ».4NPA=a2 (.PNI CA) In A's NPA and MPA ZANP = LAMP =90°,ZNPA=ZMPA=« and AP common side ANPA = AMPA “AN = AM... Ratio AM: AN 25.IF the length of a diagonal of a cube is ¥12cm, then the area of each of its faces is Solution: If a is the side of a cube, then Va is the length of its diagonal «. V8a= V1I2=2V3 3s. a=2 », Area of its face = 4cm?. Area of each face = 4 cm?. 26. The sum of the digits of the number (10009 — 20) expressed in decimal notation is Solution: 99 = 10---0- 1000° 20 = e 0-20 = 99---980 58 nines Sum of the digits 58x9+8 530 27. A point is taken on the hypotenuse of a right triangle equidistant (©) from the legs. The point divides the hypotenuse into parts 30cm and 40cm. Then 2 is equal to cmproblems and Solutions 139 Solution: PM = PN=a AP =30, PC =40, AC =70 AM _ 303 AB 70 7 {- SAMP ~ ABO} AM = V900-@ 900 — 3 000-2? +a 7 4/9002? = 30 16 x 900 — 162? = 92? 1 2527 =16x900 2? =16 x 36 =24cem 28. Two circles touch externally at C. AB is a direct common tangent touching the circles at A,B. If AC=14em, BC=48cem then the length of the direct common tangent is Solution: 4 3 LOAC = LOCA=2 RVI CNY 20,0B = ZO0,BC=y 22 = 180-ZAQC 2y: = 180- Z2BO.0 22+2y = 360-(ZAO\C + ZBO2C) = 360 — 180 = 180 (401 || BO2) sty = 90 -.ZACB is right angled at C. [. ZACB = 180 — 2 — y = 90°] “AB? = 142448? = 2?- (7? +24?) = 2x 257 = 507. AB=50.140 Problems and Solutions 29.In a quadrilateral three consecutive sides are of lengths 2,3 and 4. A circle of radius 1.2 is inscribed in the quadrilateral. The area of the quadrilateral is Solution: Cirele is inscribed in the quadrilateral ABCD AB+CD = AD+BC AD AB+CD-BC=6-—3=3 Area of ABCD = Area of AOB+Area of BOC +Area of COD+Area of DOA = PParata+gl = 6x12=7.2sq. units 30. a,b,c,d and e are positive reals. If a—2c+e = 0, b—2c-+d =0,c~2d+e=0 then the set consisting of the largest and the smallest numbers among a,b,c, d,e is — Solution: a,b,c,d,e>0 a+e=2c c lies between a and e. b+d=2c. c has to be between b and d; c+e = 2d; d has to be between ¢ and e. ate=2%=b+d . Both 6 and d lie between a and e I or a and e lie between b and d Il ‘sc+e=2d \.d lies between c and e. ics oad + > >> > | OK Not Ok. Since d is beyond ¢ and e ©. Max, Min set is {a, e}BHASKARA CONTEST FINAL TEST - 2009 1. Show that there exists no biggest number a such that a? < 2. - Solution: Let ao be the biggest number such that a9 5<2 Let C = 2-a3 and let ajay +$ iy = a+ 7 + aos = gz. 2-06) , a f5 22 = ant 16 +a (2-4 ~ a)? < 4 Coa +$(2-08) “sag >1 a2 + P= 99) 2 — of +803) aj+ ed 29) (2+ 7a”) (2= 49) < at 16 x16 a<2 1.24705 <16 = 2 a, >Oand af” <2 So f ag is the biggest mumber, then we can produce an aj > ag such that a4? <2. Such a greatest number does not exist. 2. Let ABC bea A with ZA < ZC < 90° < ZB consider the bisectors of the exterior angles at A and B each measured from the vertex to the opposite side (extended). Suppose both of these line segments are equal to AB. Find the measure of the angle A. 141142 Problems and Solutions Solution: Let a! Z bisector of exterior angle A meet CB extended at B'. Angle bisector of _ exterior angle B meets AC extended at A’. BA' = BA= AB’ But since BA= BA’, ZBAC = ZBA‘A= ZA ZABA' = 180-24 = ZABC+ZCBA' = B+- —z 7 2A+ > = 90' ZABB' = ZAB'B = 180- B. From AABB’ 360-28 + 1804 _ ig : AA 270 =3B+> B= 270-28 ie, A=540-4B (1) B 90 =2A+5 (2) Substitute (1) in (2) 90 =1080-8B+2 15B =1 -—_— : 080 -— = 155 = 990 ; B = x 2= 65x 2=192 A z= 70 — 2 x 132 A =iProblems and Solutions 143 3, Find all integers n such that 2— 12 , 2n—14 , 24m OF Solution: Given, (7m — 12)3" + 2"(2n— 14) +24n = 3"-2" 3°2" — 3"(7n — 12) -2"(an—14) = An. Adding (2n — 14)(7n — 12) we get 32" — 3"(7n — 12) — 2"(2n — 14) + (7m — 12)(2n — 14) = 24n + (2n— 14) x (7n — 12) (3" — (2n = 14))(2" — (7n—12)) = 14n? — 98n + 168 14(n? — 7n + 12) = 1(n-3)(n-4) For n=3 LHS is negative and RHS=0. Not a solution. For n= 4 RHS=0 and LHS is also 0. .n=4 isasolution. n=1,2 are not solutions. For n>5 gP414—2n = M+(3"—2n) > 14 (4) and 2*—7n+12 > n?—7+12 (5) {- 2" >n? for all n> 5} From (1) and (2) =. (3 — (an — 14))(2" — (Tn = 12)) > 14(n? — 7 + 12) 14(n — 3)(n — 4) i *. No solution for n 25. So only solutions is n = 4144 Problems and Solutions 4.2009 concentric circles are drawn with radii 1 unit to 2009 units. From a point on the outer most circle, tangents are drawn to the inner circles. How many of these tangents are of integer length? : Solution: Let P be any point on the outermost circle. Let PQ be the tangent to the circle with centre O of radius k. Then PQ? = 2009? — 4? 2009 = 7? x 41 As 41,40,9 is a pythagorean triple k= 7? x 9 and k = 72 x 40 will lead to integer values 2009? — 74 x 9? = 7? x 40 and 2009? — 74 x 40? = 7? x 9 as the integer length of tangents. No other value of k will lead to integer tangents. 5. Let H be a finite set of positive integers none of which has a prime factor greater than 3, Show that the sum of the reciprocals of the elements of H is smaller than 3. Solution: Let S= {2"-3%imn= 0,1,2,3,+++ } = set of all number in N which do not have a prime factor greater than 3. It contains 1, all powers of 2, all powers of 3;Problems and Solutions 145 and all products of powers of 2 and powers of 3. Now H CS and H isa finite set 1 1 mips or 6. The sum of the 3 concurrent edges of a rectangular block is 19cm, its volume is 144cm and its diagonal is 13cm. Find the dimensions of the rectangular block. Solution: The sum of 3 concurrent edges of a rectangular block is 19cms. If a,b,c are the 3 concurrent edges of the box, then at+b+c = 19 (a) abe = 144 (2) a+P+c = 169 (3) a+b = 19-c (atb+c)?-a?—b?-c? = 2(ab+be+ca) = 19? - 13? = 192 sab+be+ca = 96146 Problems and Solutions Divide by abe ee e¢'a bo 1 a+b 2 ; - f 1 i9-c _ 2 ctim = 3 e 1 19 er) c 73 3-19? + 96e—144 = 0 roots are c = 3,4,12 a=3,b=4,c=12 7. Let a1,@2, a3, + ,@2909 be real numbers such that a1-+a2-ta3-+ ++-azo09 = (2009)? and aj+a}-+43+-+--+a3o99 < (2009)?-+1. Show that 2008 < a, < 2010 for all ke(1,2,3,--- 2009) Solution: Let aj, = 2009+ 2, 2009 2009 2009 Sa = $5(2009+ zx) = (2009)? + S> ae ial fel ia > (2009)? 2008 Sim > 0 () k=1 Daz = > ((2009)? + 2- 2009 - xy + 2) < (2009)9 +1 2009 (.e.,) 2009 x 2009? + 2- 2009 Sax -+ S> a2 < (2009) +1 fet 2009 (ie.,) 2-209 a+ So az < 1 (2) k=lProblems and Solutions 147 ‘We need to prove that 2008 < a, < 2010 (ie.,) -1
1 then az>1 Doak +2-2009- Sa, > 1 as oz, > 0 which contradicts 2. .c1 Sa, <1 for all k= 1,2,---2009 (ie.,) 2008 < a, < 2010 for all k = 1,2,-+- ,2009 8. Prove that any prime no (22” +1) cannot be represented as a difference of two fifth powers of integers. Solution: If possible let _m,n_ be 2 integers such that m® —n? = (2" +1) But (m—n) [LHS and hence (m —n)] RHS. But RHS is a prime. om—n=1 (ie) [m=ntl n(n 418-15 = PP 41 n® + 5n4 + 10n? + 10n? + 5n+1—n° = 2" 41 2. B(n#-¢1n) + 10n?(n +1) = 2". The only prime number dividing RHS is 2. But the LHS is divisible by 5 which is a contradiction. |", There is no such representation possible.BHASKARA CONTEST SCREENING TEST - 2010 1. The number which, when subtracted from the terms of ratio a:b makes it equal to ¢:d is (A) Ses (B) a (C) stig (D) Beet Solution: ‘Terms of the ratio are a and b. Let the number subtracted be z. Then The correct answor E—%4 does not match with any of tho four options given. 2. In a Kilometer race Ram beats Shyam by 25 meters or 5 seconds. The time taken by Ram to complete the race.is . (A) 1 minute (B) 5 minutes and 30 seconds (C).3 minutes end:15 seconds -..(D)4.minutes.and 10:sccdnds Solution: Shyam covers ‘35 metets in'5 seconds. . ~. He covers I km’ = 1000 metres in 1909 x 5 = 200. sec. (i.e.,) 3 hours:20 sees. Since Ram completes 5 minutes earlier, Ram completes the race in 3 hours and 15 seconds. Answer : (C)” 3. Through Dj the mid-point of the side BC of a triangle ABC, a straight line is drawn to meet AC at E and AB produced 148Problems and Solutions 149 at F.s9 that AE = AF. Thon the ratio BF: GE is (A) 1:2 (B) 2:1 (g)13 (D) None of these Solution: Let 5” be a point on FD such that B/D = DE. BD=DC. A «. BE’ || EC and BE’ = EC. BE'CE isa parallelogram. -. In AFBE' aid AFAR. BE’ || AC. -. SFBE' ~ AFAE eB _ BB FA AE “FB=BE = EG But AE = AF BF: CE = 1. Answer : (D) None of these. 4..In the bigger of two concentric circles two chords AB sil’ AC are drawn to touch the smaller cirdle at D at. 2, Then,.BC is equal to © (A) 3DE (B) 4DE (0) 2DE = (D) 3DE “Solution: OD LAB OE LAC. i ‘where O' is.the centre of the concentric circles (tangent.is. :" to radius through point of contact) A perpendicular through thé centro to a chord bisects the chord.150 Problems and Solutions A perpendicular through the centre to.a chord biserts the chord. +1 D_is the mid point of AB and E mid point of AC. «. By mid point theorem in AABC BC =2DE Answer: (C). 5. The number of solutions of the equation 2!°s10? = 100z: is (A)0 (B)L {C) 2. * €D) 3. Solution: Pret : ‘q'810= = 1002 , “Take log to the-bese 10. Jogig® -logigz = login 1004, logge = 2+logigx Lat u=logioa. Then u?—u-—2=0. (w-Ai+1)=0 u=20r- : “fogioe = 2 ,>o=10?=100 ‘ logigt = 1 -1 =197=5 ou erat a a Answer +" (C) 6. The internal bisector AE of the'engle A of triangle ABC is (A) not greater than the median through A for all triangles. (B) not greater than the median through A for.only acute angled triangles. Pee (©) Not greater than the median through .A for only obtuse angled triangles. (D) ‘ot Jess than the median through A for. all.triangles.Problems and Solutions 151 Solution: In AABC, et AC > AB. -AE js the bisector and, AD the median. Clearly from the diagram x < y and C
c+C
.AD> AE. Answer: (A). : Aa T. Inthe adjoining diagram ABC is an OS equilateral triangle and BODE isa square. The side of the equilateral triangle is 2010. The radius of the circle is (A) 2010 (B).4020,(C).6030 (1D) 8040, Solution: Let Q. be the centre of the-circle. AO is L” to BC and DE and meets them at points and Q. ete PQ = 2010 Let r be, the radius ‘of the circle = OE =0D- 0@ = vrt— 10057 AP. = 8 42010 = 10058 OA = reOP+PA she 2010 v2 — 1008? + 100508 & Veta 00s? = 2010+1008V3—r = 1005(2+-V3)—r- Square both sides 171008? = 10057(2-+ V3)" +17 —2r(1005)(2 + v3) 2r'x 1005(2+ V3) = 100542 + v3)? +1008?152 Problems and Solutions 2r(2+ V8) = 1005(3 + d-+.4Vv3 +4) = 4020(2 + V3) + a = 2000 Answer : + (A). @ . Given a and b are integers the expression (a?+4-a-+2011)(2b+1) is (A) Odd for exactly 2010 values of a.d, (B) Odd for all values of a,b. (C) Even for exactly one value of-a and two values-of 5 (D) Odd for exactly for one value of @ and one valué of .. Solution: a? +¢=a(a+1) is evén for all integers a. a? + @+ 2011 is odd’for all values of a. 2b+1 is odd for all values of ., w. The product. -(a? + a+ 2011)(2b-+1) is odd for all values of a and b. Answer: (B). . 9: A sequence: of teal numbers-‘zq° is defitied recursively as follows. 9,71 are arbitrary positive real numbers and 2n42 = (Eat f= 0)1,2)+-* “Then the value of eyoij is {Ayr B) ap ORD) “Solution: = lta 2 = EEE Atay 144 demote ay wy tor, gece LEBER + coyltenn) | 1 Fat - Ha ay(Lbay) cayProblems and Solutions 153 a5 = te = ay = Zo = 25,21 = Xo, 22 = Zr, Fy = Te, 74 = Tg and ms = Tio = Zo. *, 22010 = Zo and 22911 = 2). Answer :” (C) 10. If zy =6 and 22y+y2x+2-+y = 63, the value of 27+? "is (A) 81 @) 18 (C) 2010 (D) 78 Solution: wytyctoty = 63 xy(oty)+(e+y) = Oe+y)+(e+y) és ‘ Wery) = 68 sty = 9 ety = (c+y)? -2xy=81-12=69, Answer: 69. It does not match any of the options given. 11. If p is the perpendioular drawn from ‘the vertex of-e regular tetrahedron we the opposite face and if each edge is equal to, 2 units, then p is (a) 8v3.(B) 8¥8 (c) 82 ~@) 88 Solution: OABC tetrahedron OA=OB=0C = AB=BC=CA=2 OG 1 ABC (Bese),154 Problems and Solutions ‘Since it-is a regular tetrahedron, ABC is an equilateral. A, and G its centroid. -.\OG 1 plane ABG,OG AG also AO = 2,°° AG = 2 medina ofthe equilateral This answer does not match any of the options given. y 32. The remainder when the polynomial 2 +25 +29 +277 + a8 4 248 ip divided by 2? 1 (A) 6z B) 2, | (O) 3 » @)1 Solution: We know that y—1 divides y%~1 for all positive integers n. Replacing-y by <7, we see that 2? — 1, divides.x?".— 1 for. all positive integers n. sy che tote = PO eta yg oy ag O-c+dc+z = xe 2 1) 0a 1) ot ge(a?®— 1) , vive sten(a® —4d) + 2(e?— 1) 462 (2? — 1), (2 — 1), ( ~1),(28 - Dy: and ‘(c?—1) are divisible exactly by‘ 2? —1. 3, This-expression Pee tenanee | 6x." Answer: (A) : 13, Consider the sequenee' “4, 4, 8,2, 0; 2,2,4,6;0,-<-!-where the nt term is the units place of the sum of the previous twoProblems and Solutions 155 terms for n = 3. If.S, is the sum to n terms.of this sequence, then the smallest ‘n’ for which 5, > 2010 is (A) 268 (B) 502 (©) 503 (D) 504 Solution: The given sequence has to repeat itself from some nt term onwards. 4,4,8,2,0,2,2, 4,6,0,6,6,2,8,0,8,8,6, 4,0,4, 4,8, 2,0,-— ‘The sequence repeats itself from the 215 term onwards (i..,) if tn isthe n“ term then to; = ti, t22 = ae +t4o = .to0, tar = ta = t; and so on. So = 80 Ss = 2-80 = 160, Sep = 3-80 = 240--- Sao = +80 2000 is a multiple of 80 2000 = 80x 25 Sao.25 = ‘Ssoo = 2000 Ssoo+3 = Ss00 + 53 = 2000 + 16 = 2016 > 2010. . 503 is the smallest. n ‘such that S,, > 2010. Answer: (C) 14. P isa point inside an equilateral, trianglé of side 2010 units. The sum of the lengths of the perpendiculars drawn from P to the sides is equal to (A) 2010 (B) 2010V3 (C) ,1005V% (D): a8 Solution: Let PX,PY,PZ be 1"’s drawn. from-P to BC,CA,AB respectively. Area of AABC = area of APBC +. area:of APCA + area of APAB*156 Problems and Solutions + (2010)? |= 32010 x PX+ ; x 2010 x PY ; x 2010 x PZ = -(1005)(PX + PY + PZ) PX+PY+PZ = WB, Mug x 10 Answer: (0) 15. The equation. loga,(2) (log, 2)? + (logaz)*= 1 has... (A) A root less than.1. (B) Has only-one root. greater ‘than’ (C) Two irrational roots (D):No real roots Solution: Wé use change of base theorem for logarithms. Joga @ = log, x «log, b Change the base to 2 where the base is not 2. Here as 2r is given as bese 2x must be >0 (i.e..) ¢>0. Joga 2 ~loggz _* 1— logy 2) 10892 (2) logs 2 ("TF loge +, The given equation’ becomes tt logy (2) “logy, 2 1—logaa 2 A T# log, = 10822)” + (logy 2)" = 1-Problems and Solutions 157 put log, silt gl ae ita” +e = 1 l-utv(1+u)) od Coreen =1 (1+u) uf(L+u)tw(l—u) = i+u wW+ut-wW+u?—u-1 = 0 w=1isaroot LHS:(u—1)(ut +2u8 tu? +2u+1) = 0 uf +203 +-u? + 2u+.1 will never become zero for u >'0. So it has no positive root. u cannot take negative values. = 1 is the only positive root. (ie.,) logge = 1 (1e.,) 2 =2 is the only positive root > 1. Answer: (B) 16. The value of 4/204 14V2-+ 3/20 14V2 is Solution: The given expression =2+ V242—/2 = 4, a8 YO + 1472 = 2+ V2 and 920-1478 =2- 2. 17. If a,’ are positive and a-+b=1 the minimum value of a4+54 fe Solution: a?+0? = (a+6)?-2ab° ont > vob AM.- G.M-inequality vob <} ox ; 2ab < 5 ee < a 20°? <5 : aoe = 1-2b>1~Z555158 Problems and Solutions af 4 bf = (a? +b)? = 2078? ; > f-8xses-sat Minimum value of af +04 is - Answer : is ; : 18. The whole surfasé area of rectangular block is 1332cm”. The length, breadth and height are in the ratio 6:5;4. The sum of the length, breadth and height is centimeters. - Solution: Let Length = 62 Breadth 52 Height = 4x Total surface area = 2(30x + 202? +2427). 2 x 74x? = 1332 Ta? = 668 ‘ w= 9 2 Sum of the length, breadth and height 3 = 60+ 5a + 4a = 152 = 45cms, Answer: 45 cms. (9. If [2[t+o+y =10,2+|yl-y = 12 then ty= Solution: If you take both 2,y positive, then equations become 2t+y=10 and t=12 y= 14 Not Possible * Both z,y <0. Then {2| = —z,|y}=—y ¢Problems and Solutions 159 y=10 Again impossible. Ifz>0 and y<0.then wey = 10 =2. ae z~2 gy =2-19-22_ y= poems HB 20. Two perellel sides of a trapezoid are 3 and 9, the non ‘parallel sides are 4 and 6. A line parallel to the bases (parallel sides) divides the trapezoid in to two trapezoids of equal perimeters. The ratio in which each of the non-parallel sides is divided.is Solution: Let AP :PD=A:1-2 AP=4i BQ=6 PD=4(1-A) QC=6(1—-A) Perimeter of APQB =Perimeter of PQCD ie3+4+64+PQ = 4(1-)+601=\) 494 PQ -20X = 19+3=16 wah aa 4 1 A= g InAs ce AP + PD = 431 PD: AP = 1:4 Answer : It is either 1:4 or 4:1.160 Problems and Solutions 21. Triangle ABC has AB = 17,AC = 25 and: the altitude to BC has length 15. The sum of the possible values of BC js Solution: If AABC is acute angled, then, BD=8 CD=2 BC=2% If MABC is obtuse angled, then, BC =12 ">, The sum of the two values of BC is 28+12=40. Answer: 40. 22. $= B+Gr hs 48, where 0
180° «. Three 10° angles is not possible in a convex polygon). 27. If an are of circle 1 subtending 60° et the-centre, has double the length 2 the arc subtending 75° at the centre in circle 2, Solution: Let r, and rg be the radii of circle 1 and circle 2. (if k=3, then y= : 60 a of = = = arc of circle 1 a0 * 202 3 ; _ 6 = Sara are of circle 2 = x 2arg = are ofcircle1 = 2% arc of circle 2 m — Wr nls Se ae rT. 2 ares of cirdle1 _ 25 area ofcircle2 ~ 4 Answer: 28. A two digit number is equal to the sum of the product of its digits and the sum of its digits. Then the units place of the number is —_________-_____.. Solution: Let a be the tens place and } be-the units place. a+b = a-b+a+d 92 = a:b af0 fp. Bad w Answer: 9. 29. Let f(z) be a polynomial of degree 1. If f(10)— f(6)= 15, then £(20) — f(5) equals164 Problems and Solutions Solution: f(e) =ac+b f(10) — f(5) = 10a-+b—(5a +d) = So=15 a=3, Then f(20)— f(5) = 45 Atiswet : 45, 30. The number of perfect-squate divisors of the number 12! is Solution: 12} = 240. 38.52. 71.441 Perfect square divisors are 27, 28, 28, 28, 210, 32, 34, 5? ‘products of even powers of 2 and even powers of 3 or 5-15 such numbers. . Products of even powers of 3 and even powers of 5. -2. Also 37-5?-2!, 1 2,4,6,8,10 5 numbers, and 34-57-2! 1 = 2,4,6,8,10 5 numbers. ‘Totally 5241415424545 = 35. Answer: 35. (If we include 1, it is 36).BHASKARA CONTEST FINAL TEST - 2010 1, Find all the natural numbers n such that n+S(n)+S(S(n)) = 2010 where S(n) = sum of the digits of n. [Example: n = 238. $(n) = 13 and $(S(n)) = 4] Solution: 2010 = 3(mod 9) If n = k(mod 9) 0 < k <9, then S(n) = k(mod 9) and 5(S(n)) = k(mod 9) <.m+$(n) + $(S(n)) = 3k(mod 9) = 3(mod 9) ok = 1,4,7(mod 9) (1) ll A number 7 < 2010 with the greatest S(n) is n = 1999, with S(n) = 28. +, Maximum values of $(n) and S(S(n)) are 28 and 10 respectively. n > 2010 — 28 — 10 = 1972 1972 = 1mod 9 .. en = 1972 n+S(n) + (S(n)) = 1972+ 19 + 10 = 2001. n= 1975 = 4(mod 9) and n+S(n)+S(S(n)) = 1975+22+4 = 2001. n= 1978 = 7(mod 9) and n+S(n)+S(S(n)) = 1978+25+7 = 2010. For n= 1981, the value of the expression is 2010. For n = 1984, the value is 2010. For n = 1987, the value is 2019. For n = 1990, the value is 2019. 165166 Problems and Solutions For n= 1993, the value is 2019. For n= 1996, the value is 2028. For n= 1999, the value is 2037. For n = 2002, the value is 2010. For n = 2005, the value is 2019. For n = 2008, the value is 2019. The numbers are 2002, 1984, 1981, 1978. ABCD iis an isosceles trapezoid with AB||CD and circumscribed about a circle with CD < AB and BC AD. DG is drawn perpendicular to AB and GH is drawn perpendicular to DA. Prove that DA,DG and DH are respectively the arithmetic mean, geometric mean and harmonic mean of AB and CD. p__¥ i yy 40 G i Solution: Referring to the figure, DN =DJ and JA= AM. > DA=DN+AM= 3(DC + AB) = DA is the arithmetic mean of AB and CD. DG? = DA? - AG? 7 iB +CbD)?— flap -cp) (. ag = 4B=CP) = AB-CD DG = VAB-CDProblems and Solutions 167 = DG is the geometric mean of AB and CD. AAGD and AGHD are similar gD _ aD HD ~ GD a 1 _ AD__AD_ _AB+CD HD ~ GD?” ABCD 2ABCD 2AB-CD fee aes OD, => DH is the harmonic mean of AB,CD. 3. Find the set of all ordéred pair of integers (a,b) such that g.c.d (0,6) =1 and 24 1 ig on integer. b 2a 25a? + 146? 25ab ~". 25 divides numerator leading to 25|14b? Solution: is an integer. gc.d.(25,14)=1 = 25/0 + 5/d (ie.,) b=5e ged(a, 5c) = 1 ged(a,c) = 1 25a7 425-14? _ is an integer. 25a-5e (ie) erie is an integer. ac Now al(a? + 14c*) = all4c? = al14, since ged(a,c) = 1 a=1,2,7,14 @ 2 since © + ME is an intoger cla? Since ged(a,c) = 1,e=1. bss ay oe ee 15 a=1 b=5 G+ =P=8 2,7 _9 ; a=2 b=5 F+55 = g not an integer168 Problems and Solutions 7, 70 7,29 : @=7 b=5 F4+ Te = E+E = Z not an integer 4.7 1 15. b=5 Stay = 3 integer Solutions are (1,5) (14,5) ‘Two circles of radii 4 and 16 touch externally. Prove that the length of the chord intercepted on the transverse common. tangent to the two circles, by a circle of diameter D which 4D touches the given circles externally, is —. 5 Solution: Let A,B,O be the centres of the circles of radii 4,16 and = respectively. The circles with centres A,B touch externally at C’ and the transverse common tangent is D intercepted by the circle of radius > at D,B. To find DE DE = 2DP (Pis the foot of the perpendicular from O on DE) D\? =) _op2 2 (3) OP’ VD? — 4x? where OP = x. i]Problems and Solutions 169 From the figure, CF = x, where OF is perpendicular to AB. Let us calculate OF in two ways. OF* = 04? — AF? and OF = OB? — BF? => OA?-AF? = OB?- BF D 2 DENG (3 +18) —(16 +2)? = (F+4) — (4-2)? 2 2 D? > T+ GD + 256 — (256 + 2 + 32x) 2 =i F +164 4D - (16-42? ~ 82) 16D—32¢ = 4D +82 = 36D? _ 9p? 427 = Sat 100 = "25 9D? 16D? A = 2 — = yf DE = pr- 2 = 22 _ 4D feb 5.N is a natural number such that it is the product of three distinct prime numbers. Find all such natural numbers such that the sum of all its composite divisors is equal to 2N. “+1, Solution: Let n =pqr where p,q,r are 3 distinct primes. The composite divisors of n are pq,qr,rp and pgr. Given pg+qr+rp+pqr = 2pqr+1 (ie.,) pa+qr+rp = par+l. Dividing by pgr we get, } ae Sear 5 ee eter eae par This implies that the sum of the reciprocals of 3 distinct prime numbers is > 1.170 Problems and Solutions Consider p = 2,q = 3,r = 5. Then eee¢ea say 2°3°5 30 "2-3-5 cardi Ae But if 5 is replaced by a higher primer number then 5+ 5 += will be <1 and hence cannot be equal to —. :. The only par solution is p=2,q=3,r=5. (ie.,) n= 30. 6. Let ABCD be a convex quadrilateral such that ZDAB is acute. ADB and ZACB are complementary angles. ZDBC and 2ZDBA are supplementary angles. Show that (DB + BC)? = AD? + AC? Solution: Extend CB to D’ such that BD’ = BD. Let ZDBC = 8 : and ZDBA=a. Given § + 2a = 180°. .. ZABD! = 180-a-f=a. Let AB and DD’ meet at E. ™ B c In ADBE and D'BB, BE is common, BD'=BD ZD'BE=ZDBE=a -. AD'BE = ADBE (By SAS congruence) ZD'EB = ZDEB = 90° and D/E = ED. In AAED! and AAED, AE is common ZAED! = ZAED = 90° ED! = ED. AAED! = AAED AD! = AD Hence AAD'B = ADB (88S congruence).Problems and Solutions 14 Hence ZAD'B = ZADB. ZAD'B + ZACD! = ZADB + ZACB = 90° (given). AAD'C is right angled at A Dic? = AD? + Ac? (D'B+BC)? = AD? + AC? (ie.,) (DB + BC)? = AD* + AC? 7. a and b are two positive reals such that a” = a +1 and b" = b+ 3a for each integer n > 2. Find with proof which is greater a or b. Solution: If a = 1 then by a" = a+1 we have a contradiction. So clearly a#1. By AM —GM inequality, (a+1)? > 4a qa) b Assume beaswce2l (sy 0 ()"- 2% ("= b+3a,a"=a+1) : ()" . =" : @) Since a
CFB = ACR=y. Since AGF = 90°, QGF = 90 - y > COF = 90°. In ABCR, AH, CP, BQ are altitudes. «. They are concurrent. Solution. Given a= bed; b= eda; ¢= dab; d= abe where a,b, c,d are positive integers. Now a+b = bed + cda = cd(a + b) at+b=cda+b)>ed=1 as atb#0. Similarly b+c > da=1; c+d => ab bc = 1. But ab=cd=da=be=1>a4 =e=d=1 (atbtet+dt _ (1+ 1+141)¢ (ab+ be + cd + da)? (+14141)? Answer: (A).BHASKARA CONTEST SCREENING TEST - 2011 PART - A 1. n is a natural number greater than 1, and Set nea nse vat 10 | vn+13 n n+3 a n+9 n+12 1 1 cE A= then (A)A=B (B)A=2B (C)A
B Solution: For any natural number m > 1 m1 < mi ive, (m—1)(m+1) < mi m+1 i “< m m vmt1 ae 1 m ‘m—1 For any t = 0,1,2,3,4,---, : =n, VEE 1 taking m+3t=n, GEEEH < te vn+1 1 > A vnt+4 ete., : n+3 n+ Adding we get A < B. Answer: (C) 2. How many distinct rational numbers (a,b,c,d) are there with alogyo 2+ blogig 3 + clogyg 5 + dlogyy7 = 2011. (A) 0 (B) 1 (0) 5 (D) 2011 174Problems and Solutions 175 Solution: The given equation on simplification reduces to logyo(2°3°5°74) = 2011 = —gaghse74 — 19201 = 22012 . 52011 Let I be the least common denominator of a,b,¢,d => glaglbslegld = 2201M | grout Now the powers are all integers. = — la= 20111, Ib =0, le = 20111, ld=0 > a=2011, b=0, c= 2011, d=0 (a,b, c,d) = (2011,0,2011,0) is the only solution . Answer : (B) ABCD is a rectangle in which AB = 8,AD = 9. EB is on AD such that DE =4. H ison BC such that BH = 6. EC and AH cut at G. GF is drawn perpendicular to AD produced. Then GF = (A) 20 (B) 22 (C) 18 (D) 15 pees \ BINS Re eee ee Solution:176 Problems and Solutions s s From the figure we have Go _ on 28 GE EA 5 CE _ GE-GC_5-3_2 GE ~ GE 55 GF _ GE_5 CD ~ CE 2 GF 2 ee F = 2 > 3 5 > G 0. Answer: (A) . The sides of the base of a rectangular parallelepiped are a and b. The diagonal of the parallelepiped is inclined to the base plane at an angle @. Then the lateral surface area of the solid is (A) 2(a-+ 6) Va? +P tan 9 (B) (@+.b)Va? +0 tan@ (©) (e2+0*)Va¥btand (D) 2(a2+6*)Va+ btan 6 Solution: ial From the figure OA vere oe tnd = aap DB ZL | where ¢ is the height. | 2 => c=Va+Ptand. 7 A Lateral surface area = 2ac + 2be = 2(a+b)Va? +6? tand Answer : (A). . The number of integers n which satisfy (n? — 2)(n? — 20) <0 is (A) 3 (B)4 (C)5 (D) 6Problems and Solutions 177 Solution: (n? —2)(n? -20) <0 > 2
n?=4,9,16 > n=4+2,43,44. ‘There are six values for n. Answer: (D). 2 In the adjoining figure, O is the circum centre of the triangle ABC. The perpendicular bisector of AC meets AB at P (A) 22PQB =3ZPBO (B) 32PQB = 2/PBO (C) 42PQB =52PBO (D) None of these Solution: R is the midpoint of AC. Join OA. Draw OSLAB. POB =9-C (fom ARCQ) AOB = 26 and SOB=C = PBO =SBO 0-6 «. PQB = PBO. Answer: (D) 7. There are 15 radial spokes in a wheel, all equally inclined to one another. Then there are two spokes which178 Problems and Solutions (A) lie along a diameter of the wheel (B) are perpendicular to each other (C) are inclined at an angle of 120° (D) include an angle less than 24° Solution: Let 2° be the angle between two consecutive spokes. Then 152° = 360° 2 = 24 Let the angle between some two spokes = 90° 2d = 90° i \ = m= $9 ¢ an integer. Similarly for 180°. Let 24m = 120° 120 _ => m= =5. 2 Thus the angle between a spoke and the 6th spoke from it include an angle 120°. Answer: (C) bad Three teams of wood-cutters take part in a competition. ‘The first and the third teams put together produced twice the amount cut by the second team. The second and the third team put together yielded a three-fold output as compared with the first team. Which of the teams won the competition? (A) first team (B) second team (C) third team (D) there is a tie Solution: Let 2,y,z be the amount of wood cut by the I, II, III teams respectively. ttz = wy 0) ytz = 30 (2) (1)-(2) gives — y = 2y — 32Problems and Solutions 179 > 4e=3y > y= = 2-8 => y=B>aondz= >a ond 2>y => Third team wins. Answer: (C). 9, The number of positive integral values of n for which (n3 — 8n? + 20n — 13) is a prime number is (A) 2 (B)1 (C)3 (D) 4 Solution: (n—1) is a factor of the given expression, since the expression vanishes for n = 1. Factorizing we get, n?—8n? +20n—13 = (n—1)(n?—7n +13). If it is a prime, 1 must be a factor and the other must be the prime number. Put n-1=1 > n=2 and n?—7n+13=3 which isa prime. If n?—7n+13=1 + n?-7m+12=0(n—3) (n—4) n=3,4. n—1=2 or 3 which are primes. Therefore, there are 3 values. Answer: (C). 10. The value of the expression ee is equal to (A) vz for all x >0 (B) -Vi for 0<2<2 (C) Vz for 0<2<2 (D) -Yz for all z>0 Solution: V@+ 2 = 8a _ vt: V(e- 2)? 2 ja — Ve a “Te-) 2) When 0<2 <2, |x—2|=—(x-2).180 Probleme and Solutions «. The expression redue For 2ifm>2 2) ifn <2 = 2 So the other options are i Answer : (B). 11. The number of positive integers ‘n’ for which 3n — 4, 4n—5 and 5n—3 are all primes is (A) 1 (B)2 (c)3 (D) infinite Solution: 3n—4+4n-—5+5n—3=12(n—1) which is even => all of them cannot be odd. => one or all must be even => one even prime and two odd primes. And 2 is the only even prime. If 3n-4=2 => |n=2 If 4n—5=2=>([n=7/4 If 5n-3=2=>[|n=1 n = 1,2. It is trivial to check that n = 2 gives all the expressions prime. Answer: (A). 12. a and b are the roots of the quadratic equation 2? + Ar — gir =0 where x is the unknown and ) is a real parameter. The minimum value of a4 + b4 is (A) 2v2 (8) 25 (C) v2 (D) 2+ V2 1 a? a+ = (a+b)? —2ab=)? + Solution: a+b=-A — ab=— 1 » 2 44 pf 2 2 1 1 at+bt = (a? +6?) = matt = (2243) ayaProblems and Solutions ae4 ¢ 1 Cg M+ 4+2- 90 1 (+ a) +2. => M42 v2 ue SBS at + bt > 24 V2 Min of at +b4=2+ V2. Answer: (D) 13. When b > 0, then 12a?b3 — a6 — b? (A) always is less than or equal to 64 (B) always greater than 64. (C) always negative (D) always lies in the interval [60, 64] Solution: Let x = a? y= B 2>0 and since b>0 y>0 J(z,y) = Lay — a8 — y8 Ofte 3 7 12-30 of 2 p— = 12¢ — 3y’ oy af o_o By 707 Be ads to (z,y) = (0,0) and (2,y) = (4,4) , 2 solutions. At (x,y) = (4,4)182 Problems and Solutions 2 Since ee is negative, and the determinant of the Hessian matrix is positive, f(x) has a maximum at (4,4). The maximum value is 64. Hence answer is (A). 14. There are three natural numbers. The second is greater than the first by the amount the third is greater than the second. The product of the two smaller numbers is 85 and the product of the two larger numbers is 115. If the numbers are z,y,z with ©
2,y,z are in AP, Let c=a—d y=a; at+d then a(a—d) = 85 a(a+d) = 115 => @-ad = 8 @+ad = 115 2a? 200 a Numbers are 8.5, 10, 11.5 2a +y +82 =17+10+92= 119. Answer : (B). 15. The number of digits in the sum 100+100+1003+- - -+10020!" is (A) 4023 (B) 4022 (C) 4024 —_(D) none of these 100 + a=10,d=15Problems and Solutions 183 Solution: Let t, = 10, k =1,2,--- ,2011 te + tk41 = 101000---00. 2k zeros, Number of digits of t; + tp =5. Number of digits of t; + t2 + tj = 1010100 is 7.. sti tate ty = 101010 100 has 2k+1 digits. (10 blocks) Hence t +----+ tao, has 2x 201141. i.e., 4023 digits. Answer: (A) PART -B 1. In the sequence a},a2,-++ ,ay, the sum of any three consecutive terms is 40. If the third term is 10 and the eighth term is 8 then the 1000th term is Solution: @+a,+a3 = 40 (1) a2 +03+a4 = 40 (2) a3+ay+a5 = 40 (3) a4+05+05 = 40 (4) (1) and (2) = a1 = a4 (2) and (3) = a2 =a5 (3) and (4) a3 =a6 So we have, ay = a4 = a7 = 010 “= Ganda 2 = a5 = ag = yy = 00 = Q3n42 = 8 (given) 3 = ag = a9 = ++ = a3n = 10 (given) ay + a2 +03 = 40 40 — 10-8 = 22 1990 = a1 = 22. Answer: 22. ay184 Problems and Solutions 2. ABCDEF is a non-regular hexagon where alll the six sides touch a circle and all the six sides are of equal length. If ZA= 140° then ZD = ——. Solution: ABCDEF is non-regular hexagon. P,Q, R,S,T,U are points of contact of the circle with the sides. AP = AQ, CT=CU BP = BU, DT=DS ES =ER, FR=FQ «. ZQOA = ZAOP = 20°. ai P 8 Now as all side lengths are equal, AP + BP = BU+UC and BP=BU. ». AP=UC = AP =AQ=CU =CT =ES=ER and BP = BU =DT =DS=FR=FQ ZPOU = ZTOS = ZROQ = 2a say. 6a+6%x20 = 360 6a = 240 a = 40 ZPBU = ZTDS = ZRFQ = 180 — 80 = 100°. ZD = 100°. Answer: 100°. 3. The difference between the largest 6 digit number with no repeated digits and the smallest six digit number with no repeated digits is *Problems and Solutions 185 Solution: Largest six digit number is 987654 Difference 885309 Answer : 885309. 4, Three consecutive integers lying between 1000 and 9999, both inclusive, are such that the smallest is divisible by 11 and the middle one by 9 and the largest by 7. The sum of the largest such four digit numbers is Solution: Let n be the middle number. Then 9 divides n. =1 (mod 11) and =6 (mod 7) . n= 9k where 112 << 1111 and k must be as large as possible. 9k —1 = 11k — (2k +1) is divisible by 11. 2k +1 is divisible by 11. Also 9k +1= (2k +1) mod 7 9k +1 is divisible by 7 = 2k-+1 is divisible by 7. <.2k +1 is an odd multiple of 77. Since we are looking at the largest 4 digit numbers with this property, consider 77 x 27 = 2k +1 k= 2iB=1 = 1039 n= 1039 x 9 = 9351. As any two successive such numbers differ by 7x 9x 11 = 693, the next bigger number will be > 9999.186 Problems and Solutions .’. This is the largest possible n. Now n-1l+n+n+1=3n = 9351x3 = 28053. Answer : 28053. . f(z) is a quadratic polynomial with f(0) = 6, f(1) = 1 and F(2)=0. Then f(3) = Solution: (2) =0. es . The quadratic polynomial f(z) = (x — 2)(ax +6) f0) = -%=6 + b=-3 fQ) = (-e+8)=1 => atb=-1 4 a=2 J(@) = (e©-2)(2e-3) £8) = (8-2)(6-3) Answer : 3. 2 . While multiplying two numbers a and b Renu reverted the digits of a two digit number and obtained the product to be 391. Renu realized that she made a mistake as her correct answer must be even. The correct product is ——., Solution: 391 = 17 x 23. If the product is even then Renu has reversed 23 and not 17. The original product is 17 x 32 = 544. Answer: 544. 7. In a chess tournament players get 1 point for a win 0 for a loss.and } point for a draw. In a tournament where every player plays against every other player exactly once, the top four scores were 55,43,4 and 21. The lowest score in the tournament was ——.Problems and Solutions 187 Solution: If n players play, the number of games played is 2-1) Bach game is either decisive (1 +0) or draw Peo ., n(n—1) (3 + 3) . So total score is Seageats In the given data the sum of top four scores is 16; and the remaining scores are less than or equal to 23. So number of players is 7 and total score is 21. So the remaining 3 players’ total score is 43 . So, even if all have equal score it will be 1} . Lowest score cannot be 2. It can be 1}, lor }. Note: The question must be: “if all the scores” were distinct, then the lowest score was ——? Then the answer is 1 as the remaining three scores will be 2,1},1. 8. When z is real, the greatest possible value of 10* — 100” is Solution: 107-100" = 10-10 —(107* — 10 elo (10* — 5) I +2 The greatest value of the expression is . Answer: }. 9. The diagonals of a convex quadrilateral are perpendicular. If AB = 4, AD =5,CD =6, then length of BC is Solution: Oo Let AC and BD meet at fi : eet ZC | L t PB=yA % PC z 4 PD = w. B168 Problems and Solutions ety = 16 0) e+w? = 25 (2) 2+w? = 36 (3) Pog = 1 (4) (from (2) and (3)) pro 27 ©. (from (1) and (4)) BO? = 27 = BC=3v3 Answer 3V3. 10. Three circles, each of radius one, have centres at A,B and C Circles A and B touch each other and circle C touches AB at its midpoint. The area inside circle C and outside circles A and B is y Solution: M mid point of AB MC 1 AB MA=MB=MC=1. From the figure, AMCP is a square of side 1. The area excluded from the 3rd circle = 4[jr-?-3 = 49-2 ©. Shaded area =m — (7 — 2) =2. Answer: 2. 11, The number of rectangles that can be obtained by joining four of the 11 vertices of a 11-sided regular polygon is ——.Problems and Solutions 189 Solution: Since we have odd number of vertices no diagonal of the polygon will pass through the centre. For a rectangle to be formed, the 2 diagonals must be equal and bisect each other. It is not possible in a 11 sided polygon. .". Number of rectangles formed is zero. Answer: 0. 12. Let D,E,F be the midpoints of the sides BC, CA and AB respectively of triangle ABC. AB = 16,BC = 21 and CA=19. The circum-circles of the triangles BDF and CDE cut at P other than D. Then ZBPC = ——. Solution: ZBPD=ZBFD=ZBAC °; FD\|AC ZCPD = ZCED=ZCAB °; ED\|AB -. ZBPC =2ZBAC. Answer: 2ZBAC. 13. Let x and y be two distinct three digit positive integers such that their average is 600. Then the maximum value of = is v Solution: 242 = 600 = x+y =1200 x and y are 3 digit numbers190 Problems and Solutions *, Largest such « = 999. y = 1200 — 999 = 201. /. Maximum value of 2 = $97 = Answer: 33, 14. Let N = 101010---101 be a 2011 digit number with alternating 1’s and 0’s. The sum of the digits of the product of N with 2011 is 5 Solution: The product will have 2014 digits with the leading 2 digits as 20 and trailing 3 digits 111 It is 20 31 31---313 111 31 block is repeated 1004 times and we have an extra 3. Sum of the digits = 1004x4+3+2+0+1+1+1 = 4024, Answer: 4024. 15. 21, 91,22, ye are real numbers. If zj+23 <2 and y2+y3 <4, the maximum value of the expression x1y1 + zayo is Solution: (1-1)? + (v2)? 20 22 + y? — 2xy, + 2} + y3 — 2eay2 > 0 tat ity > Xen +2) = Aeiytry) < zit+aht+yi+y < 244 => ayntoy < 3. .. Maximum value of 21y1 + cay2 is 3. Answer : is 3.BHASKARA CONTEST FINAL TEST - 2011 1. If @ = 20112, 6 = 2010, ¢ = (2010 + 2011)10+2011 | and = 2011, find the value of lari, 4 _acled 4 (amet | Solution: The given expression can be written as be(a + d)(c —b) + ac(b + d)(a ~ ) + ab(c+d)(b— a) (a= b= e)(e—a) ahel(o~ 8) + (a= 2) + (b= a)] + abefe— 8) + acla—o) + ab(6~a)] (@=b)(6— (ea) adlbe(o— b) + ca(a ~ ¢) +ab(b—a)} —(a=Bb=ale=a) d+(a-B)(b—c)c(c-a)} ~(a-B— eee) = 2011- [Standard Identity: Lab(a—b) = -(a—B)(b—o)(e-a). Proof Put a=6 in LHS LHS = be(b—c) + be(o— b) = 0 ©. (a= 8) is a factor. By cyclic symmetry (6-0), (c—a) are factors. Y(a— dab = k(a—b)(b—c)(e—a). Coefficient of a in LHS= 1. Coefficient of ab in RH! -k=1 Hence the identity is proved.] 2. The internal bisectors of angles A,B,C of triangle ABC meet the circumcircle respectively at P,Q,R. I is the incentre of 191192 Problems and Solutions ABC. PQ meets BC,CI and CA at T,Y,L respectively, PR meets BC,BI and AB at S,X,M respectively. Prove (i) I is the orthocenter of the triangle PQR. (ii) RETS,RQYX and RQLM are cyclic quadrilaterals. Solution: To prove RQTS is a cyclic quadrilateral. QRC = QBC = & (angles in the same segment QC’) PRC = PAC B 2 A (angles in the same segment PC) fio = 448 SR = 438 BOP = BAP =A (angles in the same segment) A In AQBT QTC = 444 (exterior angle). => SRQ=QTC > ROQTS is acyclic quadrilateral. In the diagram, PNQ = 180° - [ROP + QPA] = 180° - (4 + 2 + 9) = 180° - 90° = 90° (since ROP = ROB + BOPProblems and Solutions 193 = ROB + BAP => PN is the altitude of APQR. Similarly, RY,QX are altitudes. ’. I is the orthocentre of APOR. «. IXPY is cyclic. = IPX=IPx =$ => RYX =$=ROB= ROX. = RQYX is acyclic quadrilateral. Note that XY||BO since IY xX = 2 =16B. Let us prove that MIL is a straight line. MIAR is cyclic since MAI = BAP = PAc = Pho = Mar. vIMA = IRA=CRA=CBA=B MIP A+ 1am = B+4 Similarly, LTP =C +4. MTP + LTP = 180° 2. MIL isa straight line. I LMP = IMP =IAR(MIAR cyclic) « Ae ©. RQLM is a cyclic quadrilateral. 3. Let A= {a? + 4ab + 6|a,b are positive integers }. Prove that 2011 4 A.194 Problems and Solutions Solution: Let for some positive integers a,b @+4ab+0? = 2011 (a+)? + 2ab 2011 => (a+b)? = 2011-2ab (*) i} RUS is odd. > (a+b)? isodd > a+b is odd = a,b are of opposite parity. => 2ab is a multiple of 4. Let 2ab = 4p (1) where p is an integer. * = (2g4+1)? = 2011-4. 4 +4g+1+4p = 2011 4q?+4q+4p = 2010 2 2010 Ptqtp = = LHS is an integer but RHS is not an integer. . 21 A. 4, If 1 < @ < 64, find the greatest value of the expression. (log 2)* + 12(logy 2)? logy (2). Solution: Let S = (log, x)* + 12(logg x)? logy (8) Ss (loga «)* + 12(log, «)? [logs 8 — logy «] => (logg x)* + 12(logy x)?[3 — logy 2] Ml WV Let logy =y- Since l
CP=atb+e BN? + DQ? -2AM -CP = (a+b)? +(a+c)? —2a(a+b+e) P42 = BE? + DF? BE? + EA? = BA? = area of the square ABEA= AAFD. ,y,2 are real numbers such that’s +y+.z = 3 and cy + yz + 20 =a (where a is a real parameter). Determine the value of ‘a’ for which the difference between the maximum and minimum possible value of « is equal to 8. Solution: ytz = 3-2 ye = a-2(3-2) ‘The quadratic equation whose roots are y,z is P-ty+z)+yz=0 (ie) 2 -1(8-2)+a-2(8-2)=0 ¥,2 are real => the discriminant of the equation is greater than or equal to zero. (ie) (8-2)? > 4(a-32 +22) => 3(x-1)? < 4(3-a) > a3 and 1-2. /1—F
a=-9,Problems and Solutions 197 7. abe is a three digit number. ab,bc,ca are two digit numbers. Determine all three digit numbers abe such that abe = ab + be+ ca. Solution: abe is a three digit number, ab, be,ca are two digit numbers. .°. a,b,c cannot be zero. As per the given condition 100a + 105 + ¢ = (10a +b) + (106+ 0c) +(10c-+a) . On simplification we get 89a = 10c+b RHS is a2 digit number. Hence, 89a is atmost 89 + a = 1 (since a is the leading digit a #0). . c+ b= 89. Now b,c are digits + b=9, c=8. ©. abe = 198. 8. If a,b,¢,d are four real numbers such that a+2b+3c+4d > 30, prove that a? +6? +c? +d? >30. Solution: a,b,c,d are four real numbers such that a + 2b-+ 3c+4d > 30. Clearly, (a — 1)? + (b- 2)? + (c-3)? + (d—4)? >0 => @+P +c 4d? —2a-4b—6e—8d+30>0 >= P4+P44+@ > 20+4+4b+6c+8d—30 2(a + 2b + 3c + 4d) — 30 2(30) — 30 30 30. Vou S P4P 4 +e VvBHASKARA CONTEST SCREENING TEST - 2012 PART- A 1. Two regular polygons of same number of sides have sides 40cm and Ycm in length. ‘The lengths of the side of another regular polygon of the same number of sides and whose area is equal to the sum of the areas of the given polygons is (in cm) (A) 49 (B) 31 (c) 41 (D) 360 Solution: Area of regular polygon = Ka? where ‘a’ is the length of a side and K depends on the number of sides. From the data we have, K(40? +9") = K(1600+81) = KC? => C=41. Answer: (C). 2. If e>y>0 and 24 = V2, the value of =H" is (A) 5 (B) 4 (©) (D) 6 Solution: =vie 2 ety zy 7 zy By componendo and dividendo we get, v2+1 _ mw _2 v2-1 0 yy z (v2+1)? = Fe Westy si aya y We I(v2+1) y gs =3-2 2 saa8 is 2442 FSV _ FY 34 ova4a-29=6. wy yo Answer: (D). 198Problems and Solutions 3. In rectangle ABCD, AB = 2BC = 4cm E and F are midpoints of AB and CD respectively. ESD and ETC are arcs of circles centred at A and. B respectively. If the perpendicular bisector line 1 of BF cuts the arcs at $ and T as in the diagram, then ST is equal to (in cm) (A) (4-2v3) (B) (3+v3) (C) (2+2V3) (D) (4v3-2) Solution: Let the perpendicular bisector of EF cut AD and BC at G,H 199 ' < <> ° « respectively G. Clearly AAGS, ABTH, are congruent right angled ahaa AS = BT = radius =2cem AG = BH GAD = lem ZAGS = ZBHT=90° GS = TH=/2-2 > GS = TH=V3em ST = GH-(GS+TH) = (4-23). Answer: (A). 4. The value of ‘a’ for which the expression {(2! -at +1)"+ (a +aé +1)" o 2Ya-2 )7 Fewer} —2!0842-2 takes the value 2012 is (A) 4048 (B) 6036 (C) 6037 (D) 4047200 Problems and Solutions Solution: Simplifying the expression we get that the given expression = 241, As per hypothesis it takes the value 2012 = 2012 a+1 = 8048 a = 8047 Answer: (D). 5. C, and C2 are two non-intersecting circles whose radii are in the ratio 1:2. A third circle cuts the smaller circle at A and B and the bigger one at C and D. AB and CD intersect at P. The ratio of the lengths of the tangents form P to the circles C, and C2 is (A) 1:4 (B) 1:8 (OC) 11 (D) 1:2 Solution: C, Q PA-PB=tt PC-PD=8 where t1,t2 are the lengths of the tangents to the smaller and bigger circles. titg=1il Answer: (C). Here the ratio of the radii of Cy, is immaterial provided they are unequal.Problems and Solutions 201 6. a,b,c are real numbers and none of them zero and E = (a +2)? + (6+ 4)? 4 (ab+ 4)?— (a+ 4) (6+ 4) (ad+ 3) . Then £ is equal to (A) 2012 when a=6=2012 — (B) 2012 when ab = 2012 (C) 4 for all real values of a and b (D) 2012 for all real values of a and b Solution: (+ ) (+3) 2+3) = (ab+ +$+2) bt Answer: (C). 7. The angles of a triangle are in the ratio 2:3:7. The length of the smallest side is 2012cm. The radius of the circum circle of the triangle (in cm) is (A) 2013, (B) 2011 (C) 4024 (D) 2012 Solution:202 Problems and Solutions ‘We have 26 + 38+ 70 = 180° 126 = 180 + 0=15° The angles of triangle are 30°,45° and 105° smallest side AC =a since ZAOC = 2(30°) = 60° a= radius = 2012cm. Answer: (D) 8. Ifa = 20125 —1005,c = —1007, then the value of a ot a bye tera aso times (A) 2012 (B) 1 (c)0 (D) (2012)8 Solution: We have a+b +c = 2012 — 1005 — 1007 = 0 > ++ =3abe. Given expression = —a3 — b3 — c} + 3abe = 0. Answer: (C) 9. ABC is a triangle with AB = 13cm, BC = 14cm, and CA=15em. AD and BE are the altitudes from A and B to BC and AC respectively. H is the point of intersection of AD and BE. Then the ratio 2B = (A) 3 ®) & (©) § (D) Solution: Let BE be the altitude AC,AS AC = 14cm rn Let EC =(7+¢) cm 1 IS AE = (7-t)cm By Pythagoras theorem, we — are D7 c AB? — AE? = BE? BC? — EC? = BE?Problems and Solutions 203 => 13?-(7-t)? = BE? = 15" -(7+t)? => (7-8)? — (7-2)? = 15? — 13? > 4x 7xt=28x2 > t=2 AE =5em EC = 9m. Consider AADC and ABEC. ZACD = ZBCE (common) ZADC = ZBEC = 90° B 7 ZDAC = ZEBC (Third angles are equal) This is because the sum of the Cc angles of any triangle is 180°. AADC and ABEC are similar triangles. Now, consider ABHD and ABEC ZHBD = ZEBC common; ZBOH = ZBEC = 90° ZBHD = ZBCE (Third angles are equal) ABHD and ABCE are similar triangles HD _CE_%m _3 > HB” CB” bem 5° ©. Correct option is (A) $ Answer: (A). 10. For how many positive integrals values of x < 100 is (3*—2?) divisible by 5? (A) 16 (B) 20 (C) 24 (D) 36204 Solution: Problems and Solutions The decimal expansion of 3° ends with 3,9,7 and 1 only. Here x stands for a natural number (+ve integer) Character of 2 A multiple of 441 A multiple of 442 A multiple of 443 A multiple of 440 3 9 7 1 3” ends with digit End digit cycle of 3° The decimal expansion of ? ends with digit 1,4,9,6,5,6,9,4,1,0 only. Here x stand for positive integer. End digit of 2 End digit of 2? End digit cycle of L cOoamrannan 1 CH ROoCaAanaoe o™ 1 9 VS ‘As LOM of 4,10 is 20, the end digit of (3 — 2?) forms a cycle of 20 numbers. Let us table as follows: ot ~N 1 4 \ \ ! CEE gs be 5Problems and Solutions 205 Positive End digit End digit End digit integer x of 37 ofa? of 3° - 2? 1 3 1 2 2 9 4 5 3 7 9 8 4 1 6 5 5 3 5 8 6 9 6 3 7 7 9 8 8 1 4 7 9 3 1 2 10 9 0 9 lL 7 1 6 12 1 4 7 13 3 9 4 14 9 6 3 15 7 5 2 16 1 6 5 17 3 9 4 18 9 4 5 19 7 1 6 20 1 0 1 21 3 1 2 22 9 4 5 This cycle repeats. .. The number of positive integral values of x < 100 for which 3 — 2? ends with digit ‘5’ or divisible by 5 is 4x 5 (cycle of 20) = 20 Answer: (B).206 Problems and Solutions 11. If one root of Ya—a+ Vb+a = a+ Vb is 2012, then a possible value of a,b is (A) (2000, 2012) (B) (4024, 2012) (C) (1000,1012) (D) (1012, 1000) Solution: Ya—a+ Vb+a=Va+vob. Squaring both sides we get, a-x+b+042/(a—a)O+a) =at+b+2Vab = 2A/(a—a)(b+a) =2Vab (a-2)(b+<) = ab ab—br+ax—2? =ab => wo +br-ar=0 a(x+b—a)=0 => £=0 or r=a-b a—b= 2012. Answer: (B) 12. In the figure shown, BD =CD, BE=DE, AP = PD and DG\ICF. pS area of AADH Then ‘area of AABc '8 equal to (A) 3 (8) 3 Og (D)Problems and Solutions 207 Solution: Let J be the point of intersection of CF and AE, as shown. A Consider AAGD D FP\|GD and p is mid point of AD. .. By converse of mid point theorem, F will mid point of AG. AF = FG (a) Now consider AFBC GD\|FC and D is mid point of BC. F B D Cc . By converse of mid point theorem G will be mid point of FB. FG=GB (2) From (1) and (2), AF=FG=GB .206 Problems and Solutions But AF + FG+GB = AB > AF =FG=GB=1AB. Consider AABD B By the mid point theorem PE||AB and PE=1AB. D Now consider AAFI and APIE, as shown. By alternate angles, £ A ZAFI = ZIPE and — ZPFAI = ZIEP ZFIA = ZPIE (vertically opp.) . AAFI and AEPI are similar triangles. é P ane EP IP TE A AE Eb IAS “EP IP IE~ Now consider AAGD FP\|GD. F 2. FI\|GH,IP||HD FI_GH_2 IP” DH” 3° j G H D DH = = DG (Area of AADH = 3 Area of AADG) (3) Area of ABGD = 3x Area of AABC since GB = } AB and BD =}BC. Area of AABD = }x Areaof AABC as BD = DC = }-BCProblems and Solutions 209 .. Area of AADG = Area of AABD— Area of ABGD =(}-})x Area of AABC). = Area of AADG = 3 x Area of SABC (4) . Area of AADH = exgx Area of AABC from (3) and (4) 1 Area of AADH _ 1 =gofAreaof AABC. -. Sage Answer: (A). 13. If a = 2012, b = 2011, c¢ = 2010 then the value of a? +b? +c? —ab—be—ca is | (A) 0 (B) 2012 (c)3 (D) 4024 Solution: a= 2012 6=2011 c= 2010. 5 Leta=b+1 c=b-1. Given expression is = (b+ 1)? + 6? + (b — 1)? — (6 + 1) —b(b— 1) — (b— 1)(b +1) = 30? 42-30 +41=3. Answer: (C). 14, AX and BX are two adjacent sides of a regular polygon. If ZABX = 1ZAXB, then the number of sides of the polygon is (A) 6 (B)7 ()9 (D) 5 Solution: From the figure we have 50 1a _ (180 x 3\° A y= 180 = 6=( 5 ) ae 3 Ina regular polygon of n sides each oe (n — 2)180 interior angle = “*—— x0 (n—2)180 _ 180x3 n 8 => 5n-10=3n + n=5, vt 3 Answer: (D). B210 Problems and Solutions 15. A rectangular block of dimensions 16x 108 units is painted. It is cut in to cubes of dimensions 1 x 1x 1. The number of cubes which are not painted at all is (A) 945 (B) 672 (©) 812 (D) 796 Solution: Number of cubes not painted =14x 8 x 6 = 672. Answer: (B). PART -B 1. The value of 5+ 2V13 + Y/5—2V13 is = —— Solution: Let a= /5+2V3 b= Y/5—2V3. We have the identity (a+0)3 =a? +B + 3ab(a +b) => (a+b) =54+2V13 +5 —2Vi3 +3. 4/5? — (2V18)2 - (a+b) = 104+3- ¥-27(a+b) 10-9 where «=a +b) 2° 4+9r-10=0 a—14+92-9=0 (e-1)(a? +241) +9(@-1) =0 (@-1)(2? +2410) =0 (e@-I[(e+ 3”? +93) =0 $4uuv HUD = =1 is the only real solution. a+b=1 Answer: a+b=1.Problems and Solutions 211 2. Triangle ABC is equilateral of side A length 8cm. Each arc shown in the —f diagram is an arc of a circle with the opposite vertex of the triangle as its centre. The total area enclosed within the entire figure shown (in cm?) Solution: Area enclosed by the figure (in cm?) = Area of sectors with centre A,B,C and radius BC or CA or AB —2 area of AABC a 60 2 vB oo = ax ereeta(4 x8 2 Ste _ 843 _ son — V5) Answer: 32(1— V3). 3. If a,b,c,d, satisfy the equations a + 7b + 3c + 5d = 8a+4b+6c+2d = 16, 2a-+6b-+4e+8d = 16, 5a+3b+7ce+d = —16 then the value of (a+d)(b+c) = Solution: Given that a+7b+3ce+5d=0 qa) 8a + 4b+ 6c+ 2d = -16 (2) 2a + 6b + 4c+ 8d = 16 (3) 5at+3b+7c+d=-16 (4) (2) + (3) — (1) — (4) = 4a+4d = -16 + 16-0+16=16 a+d=4 (5) | (1) + (2) + (3) +4) => 16(a + d) + 20(b+c) = -16 => 64+ 20(b+c) =—16212 Problems and Solutions 4, Lines Ly,LZa,Ls,-+-,L9 are distinct. All the lines L4,Lg,L12,L6 and Ly are parallel. All the lines 1y,L5,L9,L13,Li7 pass through a given point A. the maximum number of points of intersection of these 20 lines is Solution: Two lines (or) a pair of straight lines can intersect at a maximum of 1 point. One can select 725° pair of straight lines from the 20 lines Ly,L2,Ls,-+- ,L19,L20. Bach of these 190 pairs of straight lines can obtain one intersection point giving rise to maximum of 190 intersection points. But 5 of these 20 lines Ls, La, Li2,L1g and Lo are parallel. There will be no intersection point from any pair of straight lines from these 5 lines. .". We need to remove 5%4 intersection points from 190. All the lines L1,Ls,L9,L13,Ly7 pass through only one point. -. We need to replace 554 intersection points by 1. The maximum number of intersection points of these 20 lines is 190 -10-10+1=171. Answer: 171. 5. &,y,z are real numbers such that (x+y)? = 16,(y 42)? =36, (+2)? =81, z+y+z>3. The number of possible values of (x+y +z) is Solution: 2+y = 44, y+z = +6, 2+2 = +9 various combinations of 2(x+y+z) are 19,11,7,1,—1,7,—11,-19. oty+z=5,9 or Bo ctytz>3. Answer: 3. 6. ABCD is a square. A line AX meets the diagonal BD at X and AX =2012cm. the length of CX (in cm) isProblems and Solutions 213 » Solution: In the triangles ABX,CBX AB= 1B = BC A B ZABX = ZCBX = 45° BX is common. => QABX = ACBX => CX = AX =2012. [| Ae IC Answer: 2012. . A two digit number is 6 times the sum of its digits. The number formed by interchanging the digits is k times the sum of the digits. Then value of k is Solution: 10a+b=6(a+b) + 4a=5b. ‘The number formed by interchanging digits is 10b+ a 10b+a = kat) Sata = k (« + Fy) 9a = ae => k=5. Answer: 5. O is the centre of a circle of radius 15cm. M is a point at a distance of 5cm from O. AMB is any chord of the circle through M, then the value of AM x MB is Solution: Let CMOD be the diameter AM:-MB=CM-MD Dp = (CO-OM)(CO+0M) = CO?-OM? = 225-25 eT = 200cm Answer: 200.214 Problems and Solutions 9. The combined age of a man and his wife is six times the combined ages of their children. Two years ago their united ages were ten times the combined ages of their children. Six years hence their combined age will be there times the combined age of the children. The number of children they have is Solution: Let x be the combined age of man and his wife. Let there be n children and y be their combined age. As per problem, 2 = 6y () 2-4 =10(y—2n) (2) (This equation is valid only if Ten years ago is replaced by Two years ago) ©+12=3(y+6n) (3) Using (1) and (2) + 6y—4 = 10(y—2n) > 3y—2 = 5(y—2n) > 2y=10n-2 (4) Using (1) and (3) 6y + 12 = 3(y+6n) > 2y+4=y+6n > y=6n-4 (5) Using (5), (4) + 2(6n — 4) = 10n-2 6n-4=5n-1 3 n=3 Number of children =3. Answer: 3. 10. A two digit number is less than the sum of the squares of its digits by 11 and exceeds twice the product of its digits by 5- The two digit number is Solution: Let the two digit number be ab, 10a+b Given that a? +b? — (10a +6) = (1)Problems and Solutions 215 and 10a+b—2ab=5 (2) adding (1) and (2) a? +b?—2ab = 16 (a-b)? 16 a-b = 44. tl If a=b+4, then, Eqn. (2) becomes 10(+4)+b-2b(b+4) = 5 10b + 40+b-26?-8b-5 = 0 ie, —2b7+3b4+35 = 0 » 20? -3b-35 = 0 0 0 5 i 26? - 106+ 76-35 = 2b(b-5)+7(b-5) = b = 5 or -7/2. If a=b~4, we get b=5 or 9/2 since b is an integer b=5 and a=9. .:, The number is 95. Answer: 95. 11. An isosceles trapezoid is circumscribed about a circle of radius 2cm and the area of the trapezoid is 20cm?. The equal sides of the trapezoid have length Solution: Area of trapezoid = }(b+c)x4=20 > b+tc=10 (1) Doc Area of trapezoid = }c(2) + $a(2) + 30(2) + 3a(2) = 20 aq a => b+c+2a=20 (2) > 10420=20 (using (1)) 4 6 B216 Problems and Solutions 12. A triangle has sides with lengths 13cm, Idem, and 15cm. A circle whose centre lies on the longest side touches the other two sides, The radius of the circle is (in cm) Solution: 13414415 _ a. A = Ys(s—a)(s—b)(s—c) =84 = i 13 : 14 ap x xr 2 x xr +r = 56/9em. Answer: 56/9. 13. The sum of the roots of the equation 2 ¥** = (Vz) 21 Solution: a2? = gel? « > calor e=2/2 > Pa e = 2-827 =0 > o%(r-8)=0 > 2=0,2=8. The sum of the roots =8+1=9 Answer: 9. 14. ABC and ADE are two secants of a circle of radius 3cm. A is at a distance of 5cm from the centre of the circle. The secants include an angle of 30°. The area of the AACE is 10cm? . Then the area of the AADB (in cm?) is ——Problems and Solutions 217 Solution: Clearly the length of tangent from A to the circle is 4cm. By intersecting chord theorem, AB. AC = AD. AE @ =P=16 Area of ABC = 10 = SAB- AC sin30 = Ap 40 => AE-AC=40 Now AB-AC- AD- AE = 256 AB: AD- AC AE = 256 AB» AD = 256/40 Area of AABD = }- AB- ADsin30 _AB-AD_ 26 _16_8 a 4x40~ 105 Answer: =.218 Problems and Solutions 15. The value of « which satisfies the equation 57-54-56. -.5% = (0.04)-?8 ig ——— Solution: 52 .54.56...52% 52th 6+ +20 = pAl4243+—-+2) _ 52. 2nd > 5™+= — (0.04)-78 atte ¢ 1) og > 5 (35) = > a? +2-56=0 (x +8)(e—7) =0 r=T. Answer: 7.BHASKARA CONTEST FINAL TEST - 2012 1. H is the orthocenter of an acute angled triangle ABC, with circumcentre O. Let P be a point on the arc BC not containing A of the circumcircle different form B and C. Let D be a point such that AD = PC and AD parallel to PC. Let K be the orthocentre of the triangle ACD. Prove that K lies on the circumcircle of triangle ABC . Solution: Let K be the orthocentre of AACD. Since AD||PC and AD = PC, ADCP is a parallelogram ZADC = ZCPA=ZCBA= ZB Ina AABC if H is the orthocentre then ZBHC = ZBHM+ ZCHM = 90° —- ZHBM +90° —- ZHCM = ZC + ZB. A Cc (since ZHBM = ZNBC = 90° —C) = 180°- ZA. «In AADC ZAKC = 180° — ZADC = 180° - ZB ZABC + ZAKC = 180° . K lies on the circum circle. 219220 Problems and Solutions 2. Find all positive integer solution of the equation 4z*—32—1= 2y?. Solution: 43 9.1 = (x —1)(42? +42 + 1) (x — 1)(2a +1)? = 2y? 22+1 isodd. .. x—1 must be divisible by 2. *.2-1=2m say m2>1 2m(2e + 1)? = 2y? = m(2x+1)? = y? m(2(2m + 1) +1)? = y? m(4m + 3)? = y? ged (m,4m + 3) = ged(m, 3) Ifged (m,3) =1, then both m and (4m+3)? must be perfect squares. So we have infinitely many solutions m = k? z=2m+1=2 +1 k=0,1,2,4,5,7,8,-+° and y = k(8k? +7). If 3 divides m and ged (m,3) =3 then m= 3*-1 where k > 1 and ged (3,1) = We get 3* -1(3(4-3*-1-141))? = y? 3h#2.1(4. 381.14 1))? = y? k must be even and 1 a perfect square for a solution to exit. ie., 3%" is a perfect square which was not already covered in the earlier case as we had ged (m,3) =1. Solutions are (2? +1, k(8k? +7) k¢a multiply of 3 (20? 3" 41, BHU (g. g—1 1? 4 1)) (z,y) where ’>0 , k >1 integers,Problems and Solutions 221 1 ‘roms: We delete two of them say ‘a’ and ‘b’ and in their place we put only one number a+b-+ab. After performing the operation 2011 times what is the number that is left over. 3. Consider the set A of numbers 41, Solution: If a1,a2 are 2 real numbers then after such an operation the number (a + 1)(a2 +1) —1 remains. If we have three numbers a, a2,03 without loss of generality we can say that the first operation is carried out between a1, 02 giving (a; +1)(a2+1)—1 and ag as the remaining elements carrying out a second operation we get, ((e1 + 1)(a2 + 1) -1+1)(a3 + 1)-1 = (a1 + 1)(a2 + 1)(a3 +1) — 1 as the number left. Assuming that this is true for n numbers a1,-++ jan Carrying out n—1 such operations leads to the number (a1 +1) +--+ (am+ 1) —1 for n < m, we need to prove it for m+1 numbers QL, ++ 5 Om; Om » If the 1 m numbers are reduced to one number after m—1 reductions it will be (a1+1)-++(am+1)—1. Now with ams1, the reductions will be (a1+1)(a2+1)--- (am-1)(am41-+1)—1. If some k terms k < m are reduced to 1 term and m+1—k terms are reduced to one term after k—1 and m+1—k-1,--- steps (a total of m—1 steps), we end up with (a4, +1) (ai, +1) v(@i, +1) 1 and (aj; +1)+++(Ojq¢i-4 +1) —1 when these are reduced. By the mi step, we will get m II (a@+1)-1. i=l The above was the proof by induction for any n numbers 1, Qn. Our numbers are },},:++, agg for n = 2012. So the final number is222 Problems and Solutions (1410+ 44 2) +e + (1+ de) + + oe) 2 aa 84 gy BONE 2013 _ ma a 2011 2012 = 2013 — 1 = 2012 4, Seven digit numbers are formed by the digits 1,2,3,4,5,6 and 7. In each number no digit is repeated. Prove that among all these numbers there is no number which is a multiple of another number. Solution: Let there be two such 7 digit numbers a,b such that a= be where c is an integer greater than 1. For both a,b the sum of the digits is 1+24+3+4+5+6+7 = 28 and 28 = 1(mod9) a= b= 1(mod9) Modulo 9 of a = be gives c= 1(mod9). But c>1. ». c>10. ». @>10b> 10" which is a contradiction since a,b are only 7 digit numbers. 5. ABCD and a ei A'B'C'D! are two unequal squares in a plane placed aA a as in the figure D c (A'B" parallel to AB etc.) Prove that AA’40C"+BB°+DD". iz oe Solution: The square A’B’C'D! is displaced from the square ABCD with the corresponding sides parallel.Problems and Solutions 223 a CM Let x be the displacement along (say x-direction) i.e., BL =x and y be the displacement along the perpendicular direction (y-direction). i.e., D'M =y. Let a,b be the sides respectively of the squares ABCD and A'BIC'D'. 2 Now AA” = (b+y-a)? + (ata), CO” =y? + (b+2)? BB’ =(b+2)?+(b+y-a), DD" =y¥?+(a+2)? AA" +00" = (b-aty) + (atz)?+y? + (b+2) BB’ +DD" = (b-a+y)+(ataPt+y?+(b+2) = AA" +CC" = BB" +DD". ° . Find integers «,y,z such that az + yz + dary = 40 a +y? + cyz = 20 Solution: Let wz + yz + day = 40 (1) x+y? + ayz = 20 (2) (1)-2%(2) >224 Problems and Solutions a2 + yz + dey — 20? — 2y? — Qayz =0 ie, (022 — 2a?) + (y?z — 2y?) + (4ay — 2eyz) = 0 => 2°(z-2) + y?(z— 2) — 2ny(z— 2) =0 => (2-2)(0? +y? — 22y) =0 => 2=2or (x-y)?=0 (ie) c=y when z=2 (1) >(c+y)?=20 and ,y are integers and no square is 20. z= 2 cannot be a solution. > @=y (2) => 22? +207 =20 => o(2+2)=20 => 2% must be a perfect square. z+2=20 or z+2=5 > 2=18 0rz=3 > 2=41 or r=42. The solutions are (1,1,18), (—1,—1,18), (2,2,3) and (-2, -2,3). 7. There are two natural numbers whose product is 192. It is given that the quotient of the arithmetic mean to the harmonic mean of their greatest common measure and the least common multiple is 182. Find the numbers. Solution: Let <,y be the two numbers. Given: ay = 192 q) Let p be the GOM and q the LOM of x and y. Given: == => GP 0 P+q (Now pq=ay=192) .. (p+q)?=12x4x 192.Problems and Solutions 225 => pt+q=52 and pq Solving we get p=4, q=48. Let c= 4l, y=4m. ++ ay = 192 16lm=192 => Im=12. Prime factors of 12 are (3,4) and even though 1 is not a prime; (1,12) also works. @=4xX38=12 y=4x4=16 =4x1l=4 9 y=4x12=48 The numbers are 12,16 or 4,48. 8. Find all the positive integral solution of the equation aH zy 2013" Solution: 2,y must be > 2013 ‘We need to get y in terms of x 1 i 1_ 2~-2013 y 20132 2013-¢ _ 2013-2 _ 2013(x~ 2013) + 20132 — ys 7-2013 22013 2013? = 018+ S55 This means x ~ 2013 must divide 2013? exactly 2013? = 3? x 11? x 61? x — 2013 must be a factor of 3? x 11? x 612 2 — 2013 = 1,3,11,61, 3x 11,11 x 61,3 x 61,3 x 11x61 3?, 112, 61?, 3? x 11,3? x 61,3? x 11 x 61 11? x 3, 11? x 61, 11? x 61 x 3,612 x 3,612 x 11 61? x 11? x 3,3? x 11?, 3? x 11? x 61,11? x 612 11? x 61? x 3,61? x 3,61? x 3? x 11,3? x 11? x 612, 27 solutions for x and the corresponding values for y.
You might also like
AMTI GEMS Sub Junior Level 2
PDF
No ratings yet
AMTI GEMS Sub Junior Level 2
217 pages
Amti Gems
PDF
50% (2)
Amti Gems
217 pages
Abhyasa Pustakam PDF
PDF
88% (8)
Abhyasa Pustakam PDF
76 pages
NMTC VI
PDF
100% (1)
NMTC VI
134 pages
AMTI GEMS Primary 3 Gauss Contest Edited by S Muralidharan For PRMO RMO INMO IMO Math Olympiad Foundation by S Muralidharan AMTI
PDF
No ratings yet
AMTI GEMS Primary 3 Gauss Contest Edited by S Muralidharan For PRMO RMO INMO IMO Math Olympiad Foundation by S Muralidharan AMTI
75 pages
(NEWS) Everything You Need To Ace Science in One Big Fat Notebook - The Complete Middle School Study Guide (Big Fat Notebooks) by Unlimited
PDF
16% (19)
(NEWS) Everything You Need To Ace Science in One Big Fat Notebook - The Complete Middle School Study Guide (Big Fat Notebooks) by Unlimited
5 pages
Amti Gem Sub Junior Level 3
PDF
100% (1)
Amti Gem Sub Junior Level 3
95 pages
mahiAMTI GEMS Primary 3 PDF
PDF
100% (5)
mahiAMTI GEMS Primary 3 PDF
75 pages
AMTI GEMS Sub Junior Level 3 (Middle School Classes) From The Mathematics Teacher NSEJS RMO INMO IMO Olympiad Foundation PDF
PDF
100% (4)
AMTI GEMS Sub Junior Level 3 (Middle School Classes) From The Mathematics Teacher NSEJS RMO INMO IMO Olympiad Foundation PDF
95 pages
NMTC Daily Practice Problems S E S S I O N - 2 0 1 4 - 1 5
PDF
No ratings yet
NMTC Daily Practice Problems S E S S I O N - 2 0 1 4 - 1 5
5 pages
AMTI GEMS Sub Junior Level 2 (Middle School Classes) From The Mathematics Teacher NSEJS RMO INMO IMO Olympiad Foundation (PDFDrive)
PDF
No ratings yet
AMTI GEMS Sub Junior Level 2 (Middle School Classes) From The Mathematics Teacher NSEJS RMO INMO IMO Olympiad Foundation (PDFDrive)
217 pages
(Nick Arnold Tony de Saulles) Nasty Nature PDF
PDF
No ratings yet
(Nick Arnold Tony de Saulles) Nasty Nature PDF
81 pages
AMTI GEMS Sub Junior Level (Middle School Classes) Sample Questions and Solutions For Mathematics Talent Tests NSEJS Olympiad Foundation
PDF
100% (1)
AMTI GEMS Sub Junior Level (Middle School Classes) Sample Questions and Solutions For Mathematics Talent Tests NSEJS Olympiad Foundation
128 pages
AMTI GEMS Sub Junior Level 3 (Middle School Classes) From The Mathematics Teacher NSEJS RMO INMO IMO Olympiad Foundation PDF
PDF
No ratings yet
AMTI GEMS Sub Junior Level 3 (Middle School Classes) From The Mathematics Teacher NSEJS RMO INMO IMO Olympiad Foundation PDF
95 pages
Gems Junior 2 - AMTI
PDF
No ratings yet
Gems Junior 2 - AMTI
225 pages
GEMS Primary 2 Gauss
PDF
75% (4)
GEMS Primary 2 Gauss
185 pages
NMTC-2023 - Previous Year Question Papers For Class 5 and 6
PDF
No ratings yet
NMTC-2023 - Previous Year Question Papers For Class 5 and 6
10 pages
Crompton Greaves (CG Power) Single Phase Motors Catalogue and Price List 2020 PDF
PDF
No ratings yet
Crompton Greaves (CG Power) Single Phase Motors Catalogue and Price List 2020 PDF
10 pages
Purusha Suktam PDF
PDF
No ratings yet
Purusha Suktam PDF
13 pages
Amti Gems Junior Level 2
PDF
100% (3)
Amti Gems Junior Level 2
225 pages
AMTI
PDF
100% (1)
AMTI
214 pages
Low Carb On Any Budget
PDF
100% (2)
Low Carb On Any Budget
25 pages
NMTC 2014 Screening Test Paper Primary 5-6
PDF
86% (7)
NMTC 2014 Screening Test Paper Primary 5-6
5 pages
Amti Gems Sub Junior Level Middle School Classes Sample Questions and Solutions For Mathematics Talent Tests Nsejs Olympiad Foundation Pdfdrivecom 1 Compress
PDF
No ratings yet
Amti Gems Sub Junior Level Middle School Classes Sample Questions and Solutions For Mathematics Talent Tests Nsejs Olympiad Foundation Pdfdrivecom 1 Compress
128 pages
Untitled
PDF
100% (1)
Untitled
174 pages
IOQM Algebra Race
PDF
No ratings yet
IOQM Algebra Race
6 pages
Allen Prmo Geometry PDF Free
PDF
100% (1)
Allen Prmo Geometry PDF Free
54 pages
Amti Gems Junior Level 1 - OCR P50
PDF
100% (2)
Amti Gems Junior Level 1 - OCR P50
55 pages
IOQM Mock Test-1
PDF
100% (2)
IOQM Mock Test-1
5 pages
NMTC 2014 Screening Test Paper Sub-Junior 7-8
PDF
85% (13)
NMTC 2014 Screening Test Paper Sub-Junior 7-8
7 pages
NMTC Final Paper 2015 Std7 8
PDF
81% (21)
NMTC Final Paper 2015 Std7 8
4 pages
Bhavesh Study Circle: AMTI (NMTC) - 2004
PDF
100% (1)
Bhavesh Study Circle: AMTI (NMTC) - 2004
24 pages
NMTC Solution Primary Level
PDF
No ratings yet
NMTC Solution Primary Level
6 pages
Number System - NMTC - Sheet - Sub Junior - VII & VII
PDF
No ratings yet
Number System - NMTC - Sheet - Sub Junior - VII & VII
35 pages
NMTC Primary 2018
PDF
100% (1)
NMTC Primary 2018
7 pages
NMTC Final Stage JUNIOR 2014
PDF
60% (10)
NMTC Final Stage JUNIOR 2014
4 pages
FOUNDATION - MATHS - STD7 - 8 - M92 - FLOW - PLANNER - SAMPLE - PAPERS of RAJESH SIR 2022-23
PDF
No ratings yet
FOUNDATION - MATHS - STD7 - 8 - M92 - FLOW - PLANNER - SAMPLE - PAPERS of RAJESH SIR 2022-23
23 pages
NMTC 2017 (5th-6th-7th-8th-9th-10th) Answer Keys With Solution - AMANS MATHS BLOGS
PDF
No ratings yet
NMTC 2017 (5th-6th-7th-8th-9th-10th) Answer Keys With Solution - AMANS MATHS BLOGS
41 pages
O-CAPS-03 & 04 - PRMO (Class-VIII To X)
PDF
No ratings yet
O-CAPS-03 & 04 - PRMO (Class-VIII To X)
21 pages
NMTC
PDF
No ratings yet
NMTC
5 pages
AMTI (NMTC) - 2004: Gauss Contest - Primary Level
PDF
100% (2)
AMTI (NMTC) - 2004: Gauss Contest - Primary Level
29 pages
NMTC - 9 - 10 CKM Stage 1 JR
PDF
No ratings yet
NMTC - 9 - 10 CKM Stage 1 JR
27 pages
NMTC Question Paper
PDF
100% (2)
NMTC Question Paper
4 pages
Olympiad Foundation Prmo PDF
PDF
No ratings yet
Olympiad Foundation Prmo PDF
3 pages
O-CAPS-05: Pre-Regional Mathematics Olympiad (PRMO) : Olympiad-Classroom Assessment Practice Sheet
PDF
No ratings yet
O-CAPS-05: Pre-Regional Mathematics Olympiad (PRMO) : Olympiad-Classroom Assessment Practice Sheet
15 pages
NMTC
PDF
100% (4)
NMTC
11 pages
NMTC 9-10
PDF
100% (2)
NMTC 9-10
26 pages
2602 - IOQM - Class-8th To 10th - Mock Test-1.
PDF
No ratings yet
2602 - IOQM - Class-8th To 10th - Mock Test-1.
7 pages
Junior Group
PDF
No ratings yet
Junior Group
6 pages
NMTC-at-Junior-level-IX-X-Standards (Level2)
PDF
No ratings yet
NMTC-at-Junior-level-IX-X-Standards (Level2)
6 pages
Mathematics NTSE Stage 2
PDF
100% (2)
Mathematics NTSE Stage 2
27 pages
NMTC Primary P
PDF
No ratings yet
NMTC Primary P
10 pages
Junior 2
PDF
No ratings yet
Junior 2
225 pages
Junior NMTC 16
PDF
No ratings yet
Junior NMTC 16
16 pages
Class Test 05 Eng
PDF
No ratings yet
Class Test 05 Eng
3 pages
NMTC Primary 2019
PDF
100% (1)
NMTC Primary 2019
7 pages
Ioqm Mock Test - 1: Indian Olympiad Qualifier in Mathematics (IOQM) - 2020-21
PDF
No ratings yet
Ioqm Mock Test - 1: Indian Olympiad Qualifier in Mathematics (IOQM) - 2020-21
4 pages
NMTC Junior 2022 Solutions
PDF
No ratings yet
NMTC Junior 2022 Solutions
22 pages
AMTI (NMTC) - 2004: Ramanujan Contest - Inter Level
PDF
100% (1)
AMTI (NMTC) - 2004: Ramanujan Contest - Inter Level
25 pages
S Chidambaram: Haematology Good Health Package Test Name Result Unit Bio Ref - Interval Method
PDF
No ratings yet
S Chidambaram: Haematology Good Health Package Test Name Result Unit Bio Ref - Interval Method
11 pages
NMTC Screening Test 2017 Sub-Junior Level: Abracadabraabracadabra.,.
PDF
No ratings yet
NMTC Screening Test 2017 Sub-Junior Level: Abracadabraabracadabra.,.
3 pages
PRMO - Only Questions
PDF
No ratings yet
PRMO - Only Questions
3 pages
Problems For Prmo
PDF
No ratings yet
Problems For Prmo
4 pages
Rotational Motion
PDF
No ratings yet
Rotational Motion
19 pages
O-CAPS-05 - IOQM - Class 11 & 12 (16-08-2021)
PDF
No ratings yet
O-CAPS-05 - IOQM - Class 11 & 12 (16-08-2021)
9 pages
NMTC at Sub Junior Level VII VIII Standards
PDF
No ratings yet
NMTC at Sub Junior Level VII VIII Standards
6 pages
M9 - Simulataneous Linear Equations - Test - 1598698260044 - Dsaub
PDF
No ratings yet
M9 - Simulataneous Linear Equations - Test - 1598698260044 - Dsaub
1 page
Macros File
PDF
No ratings yet
Macros File
12 pages
Prmo Test 2 - PC PDF
PDF
100% (2)
Prmo Test 2 - PC PDF
6 pages
PCP Practicals 2021 - 22
PDF
No ratings yet
PCP Practicals 2021 - 22
2 pages
Basic Endgame Studies (Level 1, Set 1)
PDF
No ratings yet
Basic Endgame Studies (Level 1, Set 1)
1 page
NTSE Maths Test2
PDF
No ratings yet
NTSE Maths Test2
2 pages
Janmashtami Special
PDF
No ratings yet
Janmashtami Special
1 page
Oc PT 2 - Student Copy - (Eng)
PDF
No ratings yet
Oc PT 2 - Student Copy - (Eng)
6 pages
Answers and Explanations - NMAT OG - QA+ DI DS
PDF
No ratings yet
Answers and Explanations - NMAT OG - QA+ DI DS
41 pages
Sainik 6 Revision Test
PDF
No ratings yet
Sainik 6 Revision Test
22 pages
Module 1 - Session 2
PDF
No ratings yet
Module 1 - Session 2
20 pages